Sie sind auf Seite 1von 209

A needle tract infection following an inferior alveolar block injection would initially involve

which space?
A. Pterygomandibular
B. Sublingual
C. Submandibular
D. Lateral pharyngeal
Which of the following characteristics most readily distinguishes amelogenesis imperfecta from
dentinogenesis imperfecta?
A. Radiographic appearance
B. Hereditary background
C. Presence of blue sclera
D. Color of the teeth
E. Associated hair loss
The strength of a soldered connector of a fixed partial denture is best enhanced by
A. using a higher carat solder.
B. increasing its height.
C. increasing its width.
D. increasing the gap.
Which of the following is the most likely postoperative complication of bleaching a tooth that
has not been adequately obturated?
A. Fracture
B. Discoloration
C. Acute apical periodontitis
D. Chronic apical periodontitis

E. External cervical root resorption


Which of the following is the best method for evaluating centric occlusion on a newly placed
onlay restoration?
A. Shim stock
B. Articulating paper
C. Patient feedback
D. Occlusal indicator wa
A patient presents with continuous, spontaneous pain associated with the maxillary right central
incisor. This tooth has a large Class V composite restoration. Cold testing produces lingering
pain. There is no sensitivity to percussion or palpation. The most likely diagnosis is
A. necrotic pulp, chronic apical periodontitis.
B. reversible pulpitis, normal periapex.
C. irreversible pulpitis, normal periapex.
D. irreversible pulpitis, acute apical periodontitis.
In serial extraction procedures, concerns about the eruption sequence are usually related to the
eruption pattern of which permanent teeth?
A. Maxillary canines and first premolars
B. Mandibular first and second premolars
C. Mandibular first molars and incisors
D. Mandibular canines and first premolars
Which of the following compounds is used as an antiviral agent?
A. Amantadine (Symmetrel)
B. Novobiocin
C. Miconazole (Monistat)
D. Amphotericin B

A dentist primarily splints adjacent abutment teeth in a fixed partial denture in order to
A. improve the distribution of the occlusal load.
B. improve embrasure contours.
C. stabilize the abutment teeth.
D. improve mesiodistal spacing.
Radiographic signs of trauma from occlusion include each of the following EXCEPT one. Which
one is this EXCEPTION?
A. Hypercementosis
B. Root resorption
C. Alteration of the lamina dura
D. Alteration of the periodontal space
E. Vascular infiltration of the periodontal ligament
Which of the following represents the most common form of gingival periodontal disease in
school-aged children?
A. Juvenile periodontitis
B. Localized acute gingivitis
C. Primary herpetic gingivostomatitis
D. Necrotizing ulcerative gingivitis
To prevent cross-contamination to patients from dental operatory units, a dentist should use
handpieces and water spray hoses that are fitted with which type of valves?
A. Retraction
B. Pressure
C. Anti-retraction
D. Depressurization

A dentist inadvertently sealed a small carious lesion in the occlusal surface of a maxillary first
molar. This would most likely result in
A. arrested caries.
B. extension of caries.
C. discoloration of the tooth.
D. increased microleakage.
A Class II cavity preparation in a primary molar for dental amalgam restoration will not require a
gingival bevel, because the enamel rods in the area incline
A. facially.
B. lingually.
C. gingivally.
D. occlusally.
E. vertically.
A dentist is examining a tooth that has a probing depth of 5 mm and gingival recession of 3 mm.
The total amount of attachment loss is
A. 2 mm.
B. 3 mm.
C. 5 mm.
D. 8 mm.
Guided tissue regeneration is most commonly used to treat which of the following Classes of
furcations?
A. 1
B. 2
C. 3
D. 4

Which of the following is the LEAST likely to cause rampant caries in geriatric patients?
A. Poor oral hygiene
B. A decreased salivary flow
C. A change in the oral microflora
D. The side effects of medication
Which of the following describes the use of hydroxylapatite for alveolar ridge augmentation?
A. Hydroxylapatite is considered to be osteoinductive (bone forming).
B. Granule migration and excessive augmentation are common problems with
hydroxylapatite materials.
C. Compaction or compression of hydroxylapatite after augmentation generally does not
occur.
D. Patients with hydroxylapatite augmentation to the anterior maxilla should be without
their maxillary prosthesis for a minimum of one month.
The space for the eruption of permanent mandibular second and third molars is created by the
A. apposition of the alveolar process.
B. apposition at the anterior border of the ramus.
C. resorption at the anterior border of the ramus.
D. resorption at the posterior border of the ramus.
Major connectors most frequently encounter interferences from which of the following?
A. Lingually inclined maxillary molars
B. Lingually inclined mandibular premolars
C. Facially inclined maxillary molars and premolars
D. Bony areas on the facial aspect of edentulous spaces
In constructing a fixed partial denture for a patient, the dentist will use a hygienic pontic. Which
of the following will primarily determine the faciolingual dimension of the occlusal portion of
this pontic?

A. The length of the pontic


B. The masticatory force of the patient
C. The position of the opposing contact areas
D. The width and crestal position of the edentulous ridge
Each of the following methods can be used to control pain EXCEPT one. Which one is this
EXCEPTION?
A. Cortical depression
B. Psychosomatic (hypnosis) methods
C. Raising the pain threshold
D. Blocking the sensory pathway
E. Depression of the autonomic nervous system
Thiazides, which are used in the treatment of hypertension, may require supplemental
administration of
A. sodium.
B. chloride.
C. calcium.
D. potassium.
What would be the immediate concern of a dentist after removing the primary mandibular left
second molar of a 3-year-old child?
A. Preventing lip biting
B. Preventing a dry socket
C. Controlling postoperative pain
D. Providing space maintenance
E. Preventing postoperative swelling

Which of the following will most likely cause an autogenous tooth transplant to fail during the
third year?
A. Development of root caries
B. Ankylosis of the tooth
C. An inflammatory resorption of supporting bone
D. An idiopathic, chronic progressive root resorption
Which of the following dentifrice components is most likely to inactivate the fluoride ion?
A. Anionic detergent
B. Dicalcium phosphate
C. Polyacrylic spheres
D. Monofluorophosphate
Which of the following is the most common functional psychiatric disorder in the elderly?
A. Depression
B. Anxiety
C. Senility
D. Sociopathic behavior
E. Mania
A removable partial denture made of a base metal alloy is resistant to tarnish and corrosion
because of its
A. high hardness.
B. surface oxide layer.
C. noble metal content.
D. grain structure.
The supraspinal analgesic activity of morphine is mediated primarily through its influence upon
which opioid receptor subtype?

A. Mu
B. Kappa
C. Delta
D. Sigma
E. Epsilon
When a permanent tooth clinically emerges, how much of the root structure is most likely to
have developed?
A. 1/4
B. 1/3
C. 2/3
D. 4/5
Each of the following is a part of the initial preparation of a periodontal treatment plan EXCEPT
one. Which one is this EXCEPTION?
A. Root planing
B. Surgical pocket elimination
C. Home-care instructions
D. Occlusal adjustment
E. Extractions of hopeless teeth
Which pair of anesthetics is most likely to show cross-allergy?
A. Lidocaine - mepivacaine
B. Prilocaine - tetracaine
C. Procaine - mepivacaine
D. Procaine - lidocaine
E. Lidocaine - benzocaine

To ensure better thermal and protective insulation of the pulp during a capping procedure,
calcium hydroxide should be
A. applied to a thickness of 3.0 mm.
B. placed in all cavity preparations.
C. covered with a stronger base.
D. preceded by application of a cavity varnish.
E. preceded by application of a zinc phosphate cement.
When using an occlusal separator, a practitioner can expect which of the following types of TMJ
disorders to respond most favorably?
A. Chronic dislocation
B. Muscle spasm
C. Capsular fibrosis
D. Unilateral condylar hyperplasia
In shaping and cleansing the canal of a vital maxillary central incisor, a practitioner has
inadvertently perforated the apical foramen. This error can result in each of the following
EXCEPT one. Which one is this EXCEPTION?
A. Pain to the patient
B. Enlargement of the foramen
C. Trauma to the apical tissue
D. Necrotic tissue being forced into the apical tissues
Excessive depth of the posterior palatal seal usually results in
A. unseating of the denture.
B. a tingling sensation.
C. greater retention.
D. increased gagging.

A 12-year-old boy has a skeletal Class III malocclusion. As he gets older, which of the following
cephalometric values generally increases?
A. ANB
B. SNA
C. SNB
D. Y-axis
In therapeutic doses, digitalis acts primarily on the cardiac muscle. It does so by increasing the
A. force of contraction.
B. refractory period of the atrial muscle.
C. refractory period of the ventricular muscle.
D. rate of conduction of impulses to the muscle.
After a free gingival graft, the primary source of nutrition for the graft during the first 24 hours is
the
A. primary vascular anastomosis.
B. residual nutrients within the graft.
C. diffusion of nutrients from the underlying connective tissue.
D. diffusion of nutrients from adjacent vessels of the periodontal ligament.
The optimal concentration of fluoride for community water depends upon the
A. proportion of residents who are children.
B. temperature of the air.
C. caries rate of the total population.
What is the major difference between a Class V cavity preparation for amalgam and one for
composite resin by the acid-etch technique?
A. Depth
B. Convenience form

C. Position of retention points


D. Angulation of the enamel cavosurface margins
When removing an impacted third molar, a dentist divides (sections) the tooth for each of the
following reasons EXCEPT one. Which one is this EXCEPTION?
A. To shorten the surgical procedure
B. To minimize the amount of bone to be removed
C. To allow the tooth to be delivered to the occlusal aspect
D. To minimize exertion of force necessary to remove the tooth
The thickness of coronal dentin in primary teeth, compared with that in corresponding permanent
teeth, is approximately
A. one-fourth.
B. one-third.
C. one-half.
The Occupational Safety and Health Administration (OSHA) is concerned with regulated waste
within the office. The Environmental Protection Agency (EPA) regulates the transportation of
waste from the office.
A. Both statements are TRUE.
B. Both statements are FALSE.
C. The first statement is TRUE, the second is FALSE.
D. The first statement is FALSE, the second is TRUE.
Which of the following eruption sequences is most commonly found in the primary dentition?
A. Central incisor, lateral incisor, canine, first molar, and second molar
B. Central incisor, lateral incisor, first molar, canine, and second molar
C. First molar, central incisor, lateral incisor, canine, and second molar
D. First molar, central incisor, lateral incisor, second molar, and canine

For a complete denture balanced occlusion, the lingual cusps of maxillary posterior teeth on the
non-working side contacts which areas of the mandibular posterior teeth?
A. The facial inclines of the lingual cusps
B. The lingual inclines of the lingual cusps
C. The lingual inclines of the facial cusps
D. The central fossae
Thrombophlebitis, which occurs after intravenous administration of diazepam, is usually
attributed to which of the following substances in the mixture?
A. Benzoic acid
B. Ethyl alcohol
C. Propylene glycol
D. Sodium metabisulfite
Each of the following is a pharmacologic effect of phenothiazines EXCEPT one. Which one is
this EXCEPTION?
A. Sedation
B. An antiemetic effect
C. Alpha-adrenergic blockade
D. Potentiation of the action of narcotics
E. An anticonvulsant
A higher than average mandibular plane angle is often associated with which of the following?
A. A deep overbite and a vertical growth pattern
B. A deep overbite and a Class I division 2 malocclusion
C. A deep overbite and a long lower anterior face height
D. A vertical growth pattern and a Class II division 2 malocclusion
E. A vertical growth pattern and a long lower anterior face height

The copal resin varnish that is placed in the cavity preparation before the amalgam is condensed
provides
A. sealing of the margins for the lifetime of the restoration.
B. long-term sealing of several years duration.
C. short-term sealing of the margins.
D. no sealing of the margins.
A fracture through the neck of the condyle can result in the forward displacement of the condylar
head because of the pull of which of the following muscles?
A. Temporal
B. Masseter
C. Buccinator
D. Medial pterygoid
E. Lateral pterygoid
Which of the following is the most likely indication for splinting?
A. Primary occlusal trauma
B. Mobility with patient discomfort
C. Mobility with a decrease in tissue quality, secondary to hormonal imbalance
D. Mobility related to a unilateral "skid" from centric relation to centric occlusion
A dentist restored an endodontically treated tooth with a cast post-and-core and a metal ceramic
crown. Three months later, the patient calls and complains of pain, especially on biting. Tooth
mobility is normal, as are the radiographs. The most probable cause of pain is
A. a loose crown.
B. psychosomatic.
C. a vertical root fracture.
D. a premature eccentric contact.

How should the margins of a dental amalgam restoration be trimmed?


A. By carving along the margins with a sharp instrument that rests on the tooth surface
B. By carving from the restoration to the tooth with a sharp instrument
C. By carving from the tooth to the restoration with a sharp instrument
D. By burnishing from the tooth to the restoration until the amalgam is trimmed to the
margin
Opiates are CONTRAINDICATED for patients who have which of the following?
A. Severe head injury
B. Bronchial asthma
C. Renal dysfunction
D. Acute myocardial infarction
During experimental gingivitis, supragingival plaque undergoes which of the following changes?
A. Plaque mass decreases.
B. Plaque microflora become more aerobic.
C. Plaque microflora become more gram-negative.
D. Plaque microflora become predominantly spirochetal.
During a stepback enlargement of the canal space, one reason for recapitulation after each
increase in instrument size is to
A. maintain coronal curvature of the canal.
B. maintain the apical stop for filling with gutta-percha.
C. create a coronal funnel to facilitate filling with gutta-percha.
D. clean the apical segment of dentin filings that are not removed by irrigation.
The extrapyramidal syndrome seen with the antipsychotic agents is due to their action on the
A. cerebellum.

B. brain stem.
C. hypothalamus.
D. basal ganglia.
E. cerebral cortex.
Which of the following is the most appropriate initial treatment for a patient with HIV-associated
necrotizing ulcerative gingivo-periodontitis?
A. Debridement and antimicrobial rinses
B. Definitive root planing and curettage
C. Administration of antibiotics
D. Gingivectomy and gingivoplasty
A researcher establishes the following null hypothesis: "There is no difference in student
achievement between the programmed self -instructional strategy and a conventional lecturediscussion format." In this study, the DEPENDENT variable will be the
A. type of instruction used.
B. number of students who participated.
C. relevance of course material.
D. examination scores obtained.
Five hours ago, a 12-year-old boy fell and fractured his maxillary right central incisor at the level
of the gingival tissue. The exposed pulp is vital. For this tooth, which of the following represents
the treatment-of -choice?
A. Pulp cap
B. Extraction
C. Pulpotomy
D. Complete root-canal treatment
E. Pulpectomy and placement of calcium hydroxide

The wave length of light emitted by a light-curing unit should be checked periodically with a
dental light meter. This meter detects light only in the range of
A. 300-399 nm.
B. 400-499 nm.
C. 500-599 nm.
D. 600-699 nm.
The only local anesthetic that increases the pressor activity of both epinephrine and
norepinephrine is
A. cocaine.
B. procaine.
C. dibucaine.
D. lidocaine.
E. mepivacaine.
A cooperative 5-year-old child has a carious lesion in her primary mandibular second molar.
There is no tooth mobility, but the practitioner notes a small, draining sinus tract adjacent to the
tooth. Which of the following is the treatment-of -choice?
A. Extraction
B. Pulpectomy
C. Indirect pulp therapy
D. Calcium hydroxide pulpotomy
E. Formocresol pulpotomy
Which of the following is a principle of alveoloplasty?
A. Undercuts that interfere with the seating of the denture should be removed.
B. The remaining ridge should be as broad as possible, even if some undercuts exist.
C. Sharp bony projections need not be smooth as they will round off in a few days during
the healing process.

Question 49 of 100
A patient presents with numerous carious lesions. To restore these lesions, the dentist will place
composite resins and use a visible-light curing unit. The patient's history of which of the
following will necessitate that the dentist take extra precautions when using this unit?
A. Cataract removal
B. Myopia
C. Glaucoma
In adapting a pontic to the residual ridge, the dentist must maintain a proper biologic and
hygienic environment. Therefore, the pontic must NOT
A. be convex mesiodistally.
B. touch the residual ridge.
C. be concave faciolingually.
D. be concave in two directions.
Corticosteroid therapy for arthritis is contraindicated for a patient who also has which of the
following conditions?
A. Anemia
B. Nephritis
C. Alcoholism
D. Peptic ulcer
E. Rheumatic heart disease
The success of a pulpotomy for a primary molar depends primarily upon
A. a vital root pulp.
B. the patient's age.
C. the amount of root resorption.
D. the absence of internal root resorption.

The size of the x-ray tube focal spot influences radiographic


A. definition.
B. contrast.
C. density.
D. distortion.
Which of the following syringe materials is the most rigid and, therefore, the most difficult to
remove from the mouth?
A. Polyether
B. Polysulfide
C. Addition silicone
D. Condensation silicone
E. Reversible hydrocolloid
A patient undergoes an extraction of a permanent mandibular first molar and elects NOT to have
the tooth replaced. Over a period of time, the teeth adjacent to the extraction site are likely to
move in which of the following directions?
A. Second molar, mesially; second premolar, distally
B. Second molar, no movement; second premolar, distally
C. Second molar mesially; second premolar, no movement
Which of the following toothbrushing techniques is theoretically the most effective?
A. Charter
B. Stillman
C. Sulcular
D. Side-to-side
Each of the following describes the properties of improved zinc oxide-eugenol materials
EXCEPT one. Which one is this EXCEPTION?

A. They provide an excellent marginal seal.


B. They have a palliative effect on the dental pulp.
C. They have thermal insulation qualities that compare to those of dentin.
D. They are easily removed from the cavity preparation.
In the normal dental pulp, which of the following histologic features is (are) the least likely to
appear?
A. Cell-free zone of Weil
B. Palisade odontoblastic layer
C. Lymphocytes and plasma cells
D. Undifferentiated mesenchymal cells
A child who has a distal step in the primary dentition generally develops which of the following
molar relationships in the permanent dentition?
A. Class I
B. Class II
C. Class III
Which of the following best describes sweat gland innervation?
A. Sympathetic--adrenergic
B. Sympathetic--cholinergic
C. Parasympathetic--adrenergic
D. Parasympathetic--cholinergic
E. Adrenergic--cholinergic
When cementing a cast restoration, where should the dentist apply the cement?
A. Only to the tooth
B. Only to the restoration

C. On both the restoration and the preparation


Through the Bloodborne Pathogens Standard, the Occupational Safety and Health
Administration (OSHA) directs that the uniform clothing of office personnel be laundered at
each of the following locations EXCEPT one. Which one is this EXCEPTION?
A. Dental office
B. Outside laundry
C. Employee's home
Which of the following accounts for the main cause of failure of replanted teeth?
A. Ankylosis
B. Infection
C. Pulpal necrosis
D. External resorption
E. Internal resorption
For an extension-base removable partial denture, which of the following is the most important to
maintain the remaining supporting tissues?
A. Using stress releasing clasps
B. Preserving denture base support
C. Limiting eccentric occlusal contacts
D. Using plastic teeth
The late mesial shift of a permanent first molar is primarily the result of closure of which of the
following spaces?
A. Canine
B. Leeway
C. Primate
D. Extraction

A dentist surgically removes a maxillary first molar, fracturing a large portion of the palatal root.
The root is forced into the antrum and cannot be visualized. Adjacent teeth and bone are normal.
The appropriate approach to recover this root is through the
A. hard palate in the canine area.
B. maxillary incisive fossa medial to the canine.
C. maxillary canine fossa above the level of the premolar roots.
D. nasoantral wall above the middle concha or turbinate.
E. first molar alveolus by enlarging the antral opening.
Which of the following conclusions would be correct if, after six weeks, a pulp-capped tooth
were asymptomatic?
A. Pulp capping was a success.
B. Lack of adverse symptoms might be temporary.
C. Reparative dentin formation at the exposure site was complete.
D. Adjacent odontoblasts had proliferated to cover the site of exposure.
The primary advantage of an external splint over an internal splint is
A. increased rigidity.
B. increased retention.
C. increased durability.
D. conservation of tooth structure.
A negative response to the electric pulp tester immediately after a severe luxation of a tooth
indicates that the
A. pulp is inflamed.
B. electric pulp tester is giving a false reading.
C. pulp is necrotic and should be removed.
D. blood supply is interrupted, and the negative response may be temporary.

E. neural transmission is interrupted, and the negative response may be temporary.


The one relation of the condyles to the fossae in which a pure hinging movement is possible is
A. centric occlusion.
B. retruded contact position.
C. postural position of the mandible (rest vertical dimension).
D. transverse horizontal axis (terminal hinge position).
Shy, submissive children are often the product of parents who are
A. empathetic.
B. overindulgent.
C. overprotective.
D. under-affectionate.
A practitioner pickles gold alloy restorations by heating them to redness and plunging them into
an acid bath. This procedure can result in which of the following?
A. Oxidation of the metal
B. Porosity in the casting
C. Warpage of the restoration
D. Surface roughness of the restoration
An ameloblastoma is MOST likely to develop in the wall of which of the following cysts?
A. Radicular
B. Dentigerous
C. Nasopalatine
D. Branchial cleft
A dentist relined a patient's maxillary complete denture. This patient returned repeatedly for
adjustments of the erythematous areas on the ridge crest. Which of the following is the most
likely cause?

A. An allergy to the acrylic resin that was used for relining


B. Presence of a pressure spot in the relined impression
C. The loss of even centric relation contacts
D. A decrease in the occlusal vertical dimension
The four mandibular incisors of a patient were traumatized in an automobile accident three years
ago. Current radiographs show bone rarefaction in the region of the apices of the incisors. Which
of the following courses of treatment are indicated?
A. Test the pulp vitality of the incisors and perform root canal therapy only in those teeth
that do not respond to pulp testing.
B. Perform root canal therapy on the incisors and curet the entire area of bone involvement.
C. Extract the four incisors and construct a fixed partial denture.
D. Postpone all treatment and recheck the status of the condition periodically.
Which of the following teeth have a natural tendency to drift mesially?
A. Primary maxillary second molars
B. Permanent mandibular central incisors
C. Primary mandibular canines
D. Mandibular second premolars
E. Permanent maxillary second molars
Handpiece stones can be used primarily to sharpen which of the following operative
instruments?
A. Curved chisels
B. Enamel hatchets
C. Binangle chisels
D. Spoon excavators
Endodontic therapy is completed on a tooth with a periapical radiolucency. A marked reduction
in the size of the radiolucency can be expected in approximately

A. 1 week.
B. 3 weeks.
C. 1 month.
D. 2 months.
E. 1 year.
What dimension of the face has reached the greatest percentage of its adult size at birth?
A. Depth
B. Height
C. Width
A patient states that for almost a year now, she has had a rubbery, firm, painless nodule within
the substance of the parotid gland. The condition described is MOST likely a
A. mucocele.
B. lymph node.
C. benign mixed tumor.
D. squamous cell carcinoma.
E. sialolith with encapsulation.
Which of the following properties of a gold alloy exceeds a base-metal alloy in numerical value?
A. Hardness
B. Specific gravity
C. Casting shrinkage
D. Fusion temperature
E. Modulus of elasticity
The reduction of which of the following represents the MOST significant advantage of the acidetch technique?

A. Microleakage
B. Pulpal irritation
C. Setting shrinkage of the matrix
D. Coefficient of thermal expansion
Which of the following classes of drugs, when combined with a narcotic analgesic, is the MOST
likely to produce a fatal drug interaction?
A. Cardiac glycosides
B. Oral anticoagulants
C. Tricyclic antidepressants
D. Oral antidiabetic agents
E. Monoamine oxidase inhibitors
A dentist is resecting (amputating) a root for periodontal reasons. Which of the following
represents the usual treatment sequence?
A. Periodontal therapy followed by root canal therapy
B. Root canal therapy followed by periodontal therapy
C. Root canal therapy followed by a period of observation
D. Periodontal therapy followed by a period of observation
A patient returns 10 days following incision and drainage of a submandibular space abscess. The
swelling appears to have enlarged despite high doses of antibiotic therapy. What should the
dentist do next?
A. Insert a larger drain
B. Order a CBC with differential
C. Repeat the culture and sensitivity tests
D. Use proteolytic enzymes to allow the antibiotic to penetrate the abscess

Electron microscopic examination of the bacterial flora of necrotizing ulcerative gingivitis


indicates the presence of microorganisms within non-necrotic tissues in advance of other
bacteria. The organisms involved are
A. cocci.
B. spirochetes.
C. bacteriophages.
D. filamentous rods.
Carcinoma in situ exhibits each of the following EXCEPT one. Which one is this EXCEPTION?
A. Invasion
B. Pleomorphism
C. Hyperchromatism
D. Abnormal mitoses
E. Loss of polarity
What is the purpose of incorporating loops and/or helices into an archwire?
A. To increase anchorage
B. To lower the modulus of elasticity
C. To provide a center of rotation
D. To facilitate bodily tooth movement
E. To increase the activation range
The cusps to be restored with dental amalgam should be reduced by
A. 1 mm while forming a flattened surface.
B. 1 mm while following the original contour of the cusps.
C. 2 mm while forming a flattened surface.
D. 2 mm while following the original contour of the cusps.

Which of the following barbiturates MOST readily penetrates the blood-brain barrier?
A. Barbital
B. Phenobarbital
C. Secobarbital
D. Thiopental
E. Pentobarbital
When disease factors (pocketing, bone loss, inflammation) are equal in a younger patient and in
an older patient, prognosis is usually better in the younger patient. The distribution of the
remaining teeth can be an important factor in determining the prognosis.
A. Both statements are TRUE.
B. Both statements are FALSE.
C. The first statement is TRUE, the second is FALSE.
D. The first statement is FALSE, the second is TRUE.
Which of the following lesions has the greatest malignant potential?
A. Leukoedema
B. Lichen planus
C. Actinic cheilitis
D. White sponge nevus
Group function occlusion is characterized by having
A. non-working contacts.
B. working contacts.
C. protrusive contacts.
D. a long centric.
A dentist will perform a Moyers' mixed dentition analysis. Which of the following teeth will be
measured to predict the size of the unerupted canines and premolars?

A. Maxillary incisors
B. Mandibular incisors
C. Primary molars and canines
D. Maxillary incisors for the maxillary arch; mandibular incisors for the mandibular arch
Which of the following cranial nerves have parasympathetic activity?
A. II, III, VII, IX
B. II, V, IX, X
C. III, V, VII, IX
D. III, VII, IX, X
E. V, VII, VIII,
Currently, bonding of composite restorative materials to dentin depends on
A. difunctional coupling agents.
B. dehydration of the dentin.
C. etching with dilute phosphoric acid.
D. covalent bonding.
Each of the following side effects can occur as a result of systemic absorption of lidocaine
EXCEPT one. Which one is this EXCEPTION?
A. Increased gastric motility
B. Tonic-clonic convulsions
C. Decreased cardiac output
D. Respiratory depression
Which of the following is the primary reason for using plastic teeth in a removable partial
denture? Plastic teeth are
A. resistant to wear.

B. resistant to stains.
C. esthetically acceptable.
D. retained well in acrylic resin.
A patient has painful lesions on her buccal mucosa. A biopsy reveals acantholysis and a
suprabasilar vesicle. Which of the following represents the MOST likely diagnosis?
A. Pemphigus
B. Psoriasis
C. Erythema multiforme
D. Bullous lichen planus
E. Systemic lupus erythematosus
A 43-year-old patient who has mitral stenosis, secondary to rheumatic fever, requests extraction
of two periodontally involved mandibular teeth. Initially, the dentist should
A. premedicate the patient with cephalosporin.
B. premedicate the patient with amoxicillin.
C. premedicate the patient with ampicillin and gentamicin.
D. consult with the patient's physician to determine the antibiotic-of-choice.
Which of the following is an early colonizer of supragingival plaque?
A. Streptococcus sanguis
B. Streptococcus salivarius
C. Porphyromonas gingivalis
D. Capnocytophaga gingivalis
The MOST common location for a mucous retention swelling or mucocele is the
A. tongue.
B. gingiva.

C. upper lip.
D. lower lip.
E. buccal mucosa.
Which of the following is a nonsteroidal anti-inflammatory agent with a tendency to produce
blood dyscrasias?
A. Indomethacin (Indocin)
B. Ibuprofen (Motrin)
C. Ketorolac (Toradol)
D. Acetaminophen
E. Aspirin
In an attempted venipuncture, each of the following indicates accidental intra-arterial injection
EXCEPT one. Which one is this EXCEPTION?
A. The needle moves with pulsation action.
B. A vessel tends to collapse and obstruct.
C. Aspirated blood is bright red in color.
D. Injection of a small test dose of the agent is acutely painful.
E. Penetration of a vessel meets resistance and is painful.
In a Class V amalgam preparation for an incipient lesion, the ideal internal form of the
preparation has which of the following features?
A. The axial wall is flat.
B. The mesial and distal walls converge.
C. The occlusal and gingival walls converge.
D. The axial wall is uniformly deep into dentin.
Which of the following drugs is often administered intravenously to treat life-threatening
ventricular arrhythmias?

A. Quinidine
B. Lidocaine
C. Verapamil
D. Propranolol
In a protrusive condylar movement, interferences can occur between which posterior cusp
inclines?
A. Mesial inclines of maxillary, distal inclines of mandibular
B. Distal inclines of maxillary, mesial inclines of mandibular
C. Mesial inclines of maxillary, mesial inclines of mandibular
D. Distal inclines of maxillary, distal inclines of mandibular
When lowering the floor of the mouth of a patient, a practitioner detaches all or part of which
muscles?
A. Mylohyoid and geniohyoid
B. Mylohyoid and buccinator
C. Mylohyoid and genioglossus
D. Geniohyoid and genioglossus
E. Buccinator and genioglossus
The relative position of the maxilla to the cranial base can be determined cephalometrically by
the angle
A. SNA.
B. SNB.
C. Sn-GoGn.
D. SN-Pog.
E. ANB.

A patient presents with a bilateral, grayish-white lesion of the buccal mucosa. This lesion
disappears when the mucosa is stretched. Which of the following is the MOST likely condition?
A. Leukoedema
B. Leukoplakia
C. Lichen planus
D. White sponge nevus
E. Pachyonychia congenita
A dentist is restoring an endodontically treated posterior tooth with a pulp chamber-retained
amalgam restoration. Ideally, the dentist should place the amalgam to what depth into each root
canal to obtain satisfactory retention?
A. 1 mm
B. 2 mm
C. 3 mm
D. 4 mm
Secondary trauma from occlusion is seen in cases where
A. healthy gingival and osseous tissues are present.
B. traumatic changes are occurring in periodontal tissues of teeth with normal supporting
bone.
C. normal occlusal forces cause trauma to the attachment apparatus of teeth with inadequate
bone support.
Which of the following lesions is the MOST likely to become malignant?
A. Papilloma
B. Leiomyoma
C. Lymphangioma
D. Junctional nevus
E. Granular cell tumor

Which of the following is the best treatment for a recurrent ranula?


A. Cryosurgery
B. Electrosurgery
C. Marsupialization
D. Sublingual gland excision
The dentist seats a full gold crown on a patient's mandibular right second molar. As the patient
closes and as the teeth come into initial contact, the patient's jaw deflects to the right. Before
treatment, the patient's occlusal relationship had been stable. To regain stability, the dentist will
adjust the crown. On which incline of which cusp should the adjustments be made?
A. Inner (lingual) incline, facial cusp
B. Outer (facial) incline, facial cusp
C. Inner (facial), incline, lingual cusp
D. Outer (lingual) incline, lingual cusp
Debridement of an intraoral wound is accomplished with
A. a betadine solution.
B. 1% tincture of iodine.
C. 3% hydrogen peroxide.
D. 10% hydrogen peroxide.
Which of the following species is a usual constituent of floras that are associated with
periodontal health?
A. Campylobacter recta
B. Eubacterium nodatum
C. Porphyromonas gingivalis
D. Streptococcus gordonii
Each of the following drugs has a significant anti-inflammatory property EXCEPT one. Which
one is this EXCEPTION?

A. Aspirin
B. Cortisol
C. Acetaminophen
D. Ibuprofen (Motrin)
E. Indomethacin (Indocin)
A patient presents with an amalgam restoration fractured at the isthmus six months after
placement. The most likely cause is
A. recurrent caries.
B. inadequate depth of the preparation.
C. excessive width of the preparation.
D. premature occlusal contact.
A patient has a history of significant cardiovascular impairment. The maximum safe dose of
epinephrine that can be administered to this patient is
A. 1 cc, 1:50,000.
B. 2 cc, 1:50,000.
C. 1 cc, 1:100,000.
D. 2 cc, 1 : 100, 000.
Which of the following has been associated with localized juvenile periodontitis?
A. Cyclic eosinophilia
B. Lysis of neutrophils
C. Increased phagocytosis
D. Neutrophil chemotactic defects
Which of the following is the current drug-of-choice for status epilepticus?
A. Diazepam (Valium)

B. Phenytoin (Dilantin)
C. Chlorpromazine (Thorazine)
D. Carbamazepine (Tegretol)
E. Chlordiazepoxide (Librium)
An employee wishes to clean a dental-office operatory. To receive the greatest protection during
these procedures, this individual should wear which of the following types of gloves?
A. Utility
B. Vinyl
C. Fiberglass
D. Latex
Ameloblastoma of the mandible is similar radiographically to which of the following conditions?
A. Osteosarcoma
B. Pindborg tumor
C. Ameloblastic fibro-odontoma
D. Central giant cell granuloma
Where is the principal site of vertical growth of the mandible?
A. Condylar head
B. Sigmoid notch
C. Coronoid process
D. Alveolar process
In the treatment of localized alveolar osteitis, the primary purpose of the dressing is to
A. provide a vehicle for obtudent medication.
B. promote the growth of the adjacent epithelium.
C. prevent saliva from contaminating the alveolus.

D. stimulate the formation of a blood clot.


Under which of the following conditions will it be critical to mount a patient's casts on the true
hinge axis?
A. When the patient has a severe Class II occlusal relation
B. When the patient requires several fixed partial dentures
C. When the dentist plans to change the vertical dimension through restorations
D. When the dentist plans to fabricate dentures with high-cusped teeth on a fully adjusted
articulator
A 22-year-old female patient has a diffuse erythema of the gingiva with scattered, isolated ulcers
on the tongue and in the oropharynx. The marginal gingiva is edematous and inflamed. Her
condition is of three days' duration, is painful; and she has elevated temperature and malaise. The
MOST likely diagnosis is
A. aphthous stomatitis.
B. erosive lichen planus.
C. benign mucous membrane pemphigoid.
D. necrotizing ulcerative gingivitis.
E. acute herpetic gingivostomatitis.
Each of the following is a reason for restoring an endodontically treated posterior tooth with a
dowel or post EXCEPT one. Which one is this EXCEPTION?
A. To strengthen the root
B. To enhance retention of the core
C. To enhance the lateral force resistance
Which of the following is frequently accompanied by melanin pigmentation (cafe-au-lait spots)?
A. Osteomalacia
B. Hyperparathyroidism
C. Osteogenesis imperfecta

D. Polyostotic fibrous dysplasia


In complete denture fabrication, which of the following regulate(s) the paths of the condyles in
mandibular movements?
A. The height of the cusps of posterior teeth
B. The amount of horizontal and vertical overlap
C. The size and shape of the bony fossae and the menisci and muscular influence
D. The vertical occlusion, centric relation, and degree of compensating curve
Retention of fissure sealants is chiefly the result of which of the following?
A. Mechanical microretention
B. Adhesion of the sealant
C. Chemical bonding of the sealant with the enamel
D. Physical penetration of the sealant into the pits and fissures
On the basis of diagnostic test results, a dentist correctly classifies a group of patients as being
free from disease. These results possess high
A. sensitivity.
B. specificity.
C. generalizability.
D. repeatability.
Which of the following is necessary to allow for apically positioning a flap margin on the palatal
surface of molar teeth?
A. Use only a sulcular incision for the initial incision
B. Trim the flap margin to the proper length during the procedure
C. Avoid making the initial incision deep enough to reach bone
D. Extend vertical releasing incision beyond the mucogingival junction
Bradycardia is MOST commonly treated with which of the following drugs?

A. Atropine
B. Epinephrine
C. A diuretic
D. A potent vasodilator
The dentist plans to place a crown on a patient's maxillary canine. Altering the existing canineguided occlusion in a right-lateral excursive movement to that of a group-function will result in
which of the following?
A. Greater clearance on the left side-less potential for a non- working contact
B. Less clearance on the left side--greater potential for a non-working contact
C. No effect on the non-working side clearance
Which of the following is the LEAST effective for enhancing orthodontic anchorage?
A. Use of extraoral force
B. Use of .018 round wire in a .022 rectangular slot
C. Attachment to a greater number of teeth
D. Wider force distribution in the periodontal ligament
Which of the following best explains why drugs that are highly ionized tend to be more rapidly
excreted than those that are less ionized? The highly ionized are
A. less lipid soluble.
B. less water soluble.
C. more rapidly metabolized.
D. more extensively bound to tissue.
Radiographs of a patient's teeth reveal that the crowns are bulbous; the pulps, obliterated; and the
roots, shortened. These findings are associated with which of the following?
A. Porphyria
B. Pierre Robin syndrome

C. Amelogenesis imperfecta
D. Osteogenesis imperfecta
E. Erythroblastosis fetalis
A teen-aged patient presents with numerous proximal carious lesions that undermine the occlusal
enamel. Which of the following is the treatment-of-choice?
A. Restore involved teeth with onlays to preserve occlusion
B. Restore involved teeth as rapidly as possible using dental amalgam
C. Place the patient on a prevention regimen and delay treatment until the effectiveness of
home care is evaluated
Which of the following dimensions are compared in the transitional dentition analysis?
A. Arch width to arch length
B. Leeway space to freeway space
C. Leeway space to size of tooth
D. Space available to space required
E. The arch perimeter of the primary and transitional dentition
Which of the following groups of drugs is CONTRAINDICATED for patients who have
glaucoma?
A. Adrenergic
B. Cholinergic
C. Anticholinergic
D. Adrenergic blocking
Which of the following best explains why the dentist should provide a postpalatal seal in a
complete maxillary denture? The seal will compensate for
A. errors in fabrication.
B. tissue displacement.

C. polymerization and cooling shrinkage.


D. deformation of the impression material.
An employee has just terminated employment in a dental office. The dentist-employer should
retain this individual's medical records for how many more years?
A. 1
B. 3
C. 10
D. 30
Oral cytology smears are MOST appropriately used for the diagnosis of which of the following?
A. Pseudomembraneous candidiasis
B. Lichen planus
C. Squamous cell carcinoma
D. Benign mucous membrane pemphigoid
Why is a matrix for a Class II dental amalgam restoration extended occlusally to the cavity
preparation?
A. It serves as a guide to determine the completed restoration.
B. It allows for overfilling the amalgam.
C. It prevents escape of the amalgam during condensation.
A tablespoon contains how many ml?
A. 5
B. 10
C. 15
D. 20

Before undergoing any periodontal surgery, a patient should be controlling the accumulation of
plaque; BECAUSE, in the plaque-infected dentition after surgery, the incidence of disease
recurrence will be greater.
A. Both the statement and the reason are correct and related.
B. Both the statement and the reason are correct but NOT related.
C. The statement is correct, but the reason is NOT.
D. The statement is NOT correct, but the reason is accurate.
E. NEITHER the statement NOR the reason is correct.
The routine examination of a healthy 20-year-old man discloses a round, brown lesion 3 mm in
diameter, on his lower lip. The lesion is solitary, asymptomatic, and flat. Its duration is unknown.
Which of the following is the MOST likely diagnosis?
A. Blue nevus
B. Focal melanosis
C. Compound nevus
D. Malignant melanoma
E. Peutz-Jeghers syndrome
Low-dose aspirin therapy prevents the formation of thromboemboli by preferentially inhibiting
which of the following?
A. Phospholipase A2 in the blood vessel walls
B. Prostacyclin synthetase in the blood vessel walls
C. Thromboxane synthetase in the platelets
D. Vitamin K in the liver
Dentists often use barrier membranes (teflon membranes) to treat osseous defects in an attempt
to block which of the following?
A. Osseous regeneration
B. Connective tissue attachment

C. The formation of a long junctional epithelium


D. The coronal migration of periodontal ligament cells
A tray for a polysulfide rubber impression that lacks occlusal stops may result in
A. holding the tray for the entire time in the mouth.
B. a more prolonged period of setting in the mouth before the tray is removed.
C. an inaccurate final impression because of permanent distortion during polymerization.
D. an inaccurate final impression because of elastic distortion during removal from the
mouth.
Papillary hyperplasia is MOST frequently found in which of the following sites?
A. Hard palate
B. Soft palate
C. Anterior gingival tissue
D. Posterior gingival tissue
Which of the following anomalies can occur with a disturbance during the initiation and
proliferation stages in tooth development?
A. Size
B. Shape
C. Number
D. Calcification
The treatment-of-choice for an external inflammatory root resorption on a non-vital tooth is
which of the following?
A. Extraction
B. Surgical curettage of the affected tissue
C. Pulpectomy and obturation with gutta-percha and sealer
D. Removal of the necrotic pulp and placement of calcium hydroxide

E. Observation since it is a self-limiting process


To determine the exact form of a bony defect, it will be necessary for the dentist to do which of
the following?
A. Expose the area surgically
B. Examine in detail the gingival architecture
C. Take two radiographs using different angulations
D. Measure pocket depths from the cementoenamel junction
E. Relate the septal bone height to the cementoenamel junction
Which of the following microorganisms is the MOST commonly associated with root surface
caries?
A. Actinomyces viscosus
B. Streptococcus mutans
C. Streptococcus salivarius
D. Lactobacillus acidophilus
Which of the following sympathomimetic agents is the MOST potent bronchodilator?
A. Amphetamine
B. Norepinephrine
C. Phenylephrine
D. Isoproterenol
E. Methoxamine
A flabby, maxillary anterior ridge under a complete denture is frequently associated with
A. V shaped ridges.
B. Class II patients.
C. osteoporosis.

D. retained natural mandibular anteriors.


Which of the following is the MOST likely to cause gingival recession?
A. Occlusal trauma
B. Systemic disease
C. Toothbrush abrasion
D. Improper deflecting contour
E. Necrotizing ulcerative gingivitis
In obtaining an incisional biopsy of soft tissue, the dentist should do each of the following
EXCEPT one. Which one is the EXCEPTION?
A. Infiltrate local anesthetic solution around the intended site
B. Place a suture through the intended specimen before removing it
C. Obtain some adjacent normal tissue if possible
D. Place the specimen in saline, if 10% formalin is unavailable
When compared to other materials, which of the following is the main disadvantage of using
polyether elastomeric impression materials?
A. Tear easily
B. Stick to the teeth
C. Are much stiffer
D. Are not as accurate
E. Have longer working time
Which of the following jaw-relation records should be used for setting both the medial and
superior condylar guides on an arcon articulator?
A. Intercuspation
B. Centric relation
C. Lateral interocclusal records

D. Protrusive interocclusal records


To prevent exposure of a dehiscence or fenestration on a prominent root, the dentist's best
approach when elevating a flap is to use a (an)
A. double flap.
B. stripping procedure.
C. full thickness flap.
D. apically positioned flap.
E. partial or split thickness flap.
A dentist anticipates the possibility of a pulpal exposure of a vital, asymptomatic tooth during a
cavity preparation. In this situation, what should the dentist do with the carious material?
A. Remove the carious material laterally first and then remove completely from the deeper
areas of the cavity
B. Remove the carious material completely and then treat the tooth endodontically
C. Leave the carious material in the deeper areas, base, and restore appropriately
D. Leave the carious material in the deeper areas, temporize appropriately, and observe in
two weeks
Which of the following effects are common to pentobarbital, diazepam, and meperidine?
A. Anticonvulsant and hypnotic
B. Analgesia and relief of anxiety
C. Sedation and ability to produce dependence
D. Amnesia and skeletal muscle relaxation
The onset of action of a drug is primarily determined by the rate of
A. excretion.
B. absorption.
C. distribution.

D. biotransformation.
Which of the following is the primary etiology of gingivitis during puberty and pregnancy?
A. Plaque
B. Estradiol
C. Calculus
D. Estrogen
E. Progesterone
Which of the following soft tissue elements (fibers) are commonly associated with relapse
following orthodontic rotation of teeth?
A. Oblique
B. Diagonal
C. Horizontal
D. Supracrestal
A dentist adjusts the shade of a restoration using a complementary color. This procedure will
result in
A. increased value.
B. decreased value.
C. intensified color.
D. increased translucency.
The mucosa of the hard palate is the usual intraoral site for which of the following conditions?
A. Mucocele (most common location is the lower lip)
B. Sialolithiasis
C. Minor aphthous ulcer
D. Major aphthous ulcer

E. Necrotizing sialometaplasia
The pterygomandibular raphe is a tendon between which muscles?
A. Stapedius and stylohyoid
B. Buccinator and superior constrictor
C. Medial pterygoid and lateral pterygoid
D. Levator veli palatini and tensor veli palatini
E. Anterior belly of digastric and sternocleidomastoid
Which of the following contributes principally to the increase in height of maxillary bones?
A. Sutural growth
B. Alveolar growth
C. Apposition on the tuberosity
D. Apposition on the anterior surface
Which of the following is the MOST effective means of controlling postsurgical root sensitivity
after a periodontal surgery?
A. Plaque control
B. Applications of formalin
C. Applications of calcium hydroxide
D. Use of a desensitizing dentifrice
In which of the following locations do premalignant lesions of squamous epithelium MOST
often appear (in the U.S. population)?
A. Palate
B. Gingival tissue
C. Buccal mucosa
D. Dorsum of the tongue

E. Floor of the mouth FOM


Dental plaque is believed to adhere to teeth because
A. levans are soluble and sticky.
B. dextrans are insoluble and sticky.
C. the surface tension of plaque is low.
D. bacteria secrete a lipoprotein-cementing substance.
Which of the following adverse conditions may arise if the occlusal vertical dimension is
increased?
A. The closing muscles may become strained.
B. The opening muscles may become strained.
C. The closing muscles may become too relaxed.
D. Soreness may occur at the corners of the mouth.
Injection of a local anesthetic into an inflamed area usually has a less than optimal result. Which
of the following best explains why?
A. The prostaglandins stabilize the nerve membrane.
B. Inflammation reduces the availability of the free base.
C. The drug will be absorbed more rapidly because of the increased blood supply.
D. The chemical mediators of inflammation will present a chemical antagonism to the
anesthetic.
Which of the following is related to an enzyme deficiency and involves periodontal destruction
around primary teeth?
A. Hypophosphatasia
B. Cyclic neutropenia
C. Juvenile periodontitis
D. Papillon-Lefevre syndrome

Which of the following accounts for sudden relief of severe pain from a dentoalveolar abscess?
A. The action of bacteria-produced neurotoxins
B. The walling-off of the infection by the body
C. The neutralizing effect of tissue enzymes
D. Rupture through the periosteum into soft tissue
Loss of intermaxillary space, infection, or avitaminosis B can each cause which of the following?
A. Atrophic glossitis
B. Xerostomia
C. Angular cheilitis
D. Recurrent aphthae
E. Periadenitis mucosa necrotica recurrens
Hepatitis B virus infection commonly occurs by each of the following routes EXCEPT one.
Which one is this EXCEPTION?
A. Sexual intercourse
B. Prenatal transfer
C. Percutaneous inoculation
D. Ingestion
Each of the following is true regarding drug biotransformation EXCEPT one. Which one is this
EXCEPTION?
A. The rate may differ significantly in various animal species.
B. It primarily occurs in the liver microsomal enzyme system.
C. It usually converts a drug to its lipid-soluble, nonionized form.
D. It generally involves alterations of the chemical structure of the drug.
A corrected anterior crossbite is best retained by using

A. a bonded lingual or palatal retainer.


B. a mandibular acrylic guide plane.
C. the overbite achieved during treatment.
D. a maxillary Hawley retainer for stabilization.
Anterior open bites can be classified as a form of
A. apertognathism.
B. micrognathism.
C. retrognathism.
D. prognathism.
E. microgenia.
A dentist will reline an extension-base removable partial denture. Which of the following should
this dentist use to establish proper placement at the impression stage?
A. Finger pressure to the rests and indirect retainer
B. Finger pressure to the base area only
C. Simulated masticatory movements
D. Gentle biting force
Facial edema, cheilitis granulomatosa, and a fissured tongue characterize which of the following
syndromes?
A. Frey's
B. Melkersson-Rosenthal
C. Treacher Collins
A conservative Class II preparation for dental amalgam should have which of the following
characteristics?
A. Independent retention and resistance form for both the proximal and occlusal portions

B. Proximal retention and resistance form that depends upon a well-defined occlusal
dovetail
C. As wide on the occlusal as one-third the intercuspal distance
D. Preparation depth twice the width of the isthmus
The materials used in the walking bleach technique are sodium perborate and
A. 3% aqueous hydrogen peroxide.
B. 30% aqueous hydrogen peroxide.
C. 25% ethereal hydrogen peroxide.
D. 5% phosphoric acid.
A first order bend in an orthodontic wire is
A. a twist in the wire.
B. in the vertical plane.
C. in the horizontal plane.
D. a horizontal bend with a twist.
Changing several aspects of oral home-care behavior simultaneously is preferable to doing so
sequentially. Focusing on successive approximations to the ideal, oral home-care goal, from the
beginning, maximizes the likelihood of success.
A. Both statements are TRUE.
B. Both statements are FALSE.
C. The first statement is TRUE, the second is FALSE.
D. The first statement is FALSE, the second is TRUE.
Which of the following drugs causes the LEAST CNS depression and impairment of
psychomotor skills?
A. Diazepam (Valium)
B. Buspirone (Buspar)

C. Alprazolam (Xanax)
D. Chloral hydrate
The axial walls in an MOD cavity prepared for a cast gold onlay should
A. form acute angles with the pulpal wall.
B. form acute angles with the proximal walls.
C. diverge from the gingival walls to the pulpal wall.
D. converge from the gingival walls to the pulpal wall.
Which of the following MOST commonly causes sinus tracts in the gingival tissues of children?
A. Pericementitis
B. Periapical cyst
C. Periodontal abscess
D. Acute periapical abscess
E. Chronic periapical abscess
Which of the following is a major disadvantage of immediate complete denture therapy?
A. Trauma to extraction sites
B. Increased potential for infection
C. Impossibility for an anterior try-in
D. Excessive resorption of residual ridges
The dentist would like to compare biologic-risk estimates due to radiation damage. For this
procedure, which of the following will be the correct unit of measurement?
A. Exposure
B. Dose
C. Effective dose
D. Dose equivalent

Crevicular epithelium is one example of non-keratinized gingival tissue. Another example is


(the)
A. gingival col.
B. marginal gingiva.
C. attached gingiva.
D. outer gingival epithelium.
The nerve plexus of Rashkow is located in the
A. cell-rich layer.
B. cell-free layer.
C. central pulp core.
D. odontoblast layer.
In the eruption sequence of a primary dentition, which of the following usually occurs?
A. The first molar erupts before the central incisor.
B. The canine erupts before the lateral incisor.
C. The first molar erupts before the canine.
D. The lateral incisor erupts before the central incisor.
E. The second molar erupts before the first molar.
As radiographs are exposed, the heat of the filament determines the quantity of electrons
produced. The number of electrons, in turn, determines
A. the quantity of X rays produced.
B. the quality of X rays produced.
C. both quality and quantity of X rays produced.
On a prepayment basis, dental patients receive care at specified facilities from a limited number
of dentists. This practice plan is classified as which of the following?
A. Closed panel

B. Open panel
C. Group practice
D. Solo practice
Blade and root form implants are examples of which of the following types of implants?
A. Endosseous
B. Transosteal
C. Subperiosteal
D. Ceramic
Each of the following is an indication for a posterior maxillary osteotomy EXCEPT one. Which
one is this EXCEPTION?
A. Correction of mandibular prognathism
B. Intrusion of the tuberosity, preparatory to denture construction
C. Closure of the anterior open bite in some cases of apertognathia
D. Skeletal arch width discrepancies that require lateral positioning of posterior segments
Two months after an extraction, a patient becomes ill with hepatitis. The dentist may be at fault if
he
A. does not wear a mask.
B. does not use an antiseptic before injection.
C. does not wear gloves when performing surgery.
D. uses cold sterilization for his surgical instruments
Following immediate denture surgery, the best suture technique to use would be
A. continuous.
B. interrupted.
C. vertical mattress.

D. never use sutures.


E. horizontal mattress.
The blade type dental implant is most useful for
A. single tooth replacement.
B. anterior maxillary edentulous areas between first bicuspids.
C. edentulous posterior areas without posterior abutment teeth.
D. rehabilitation of a totally edentulous ridge (especially the mandible) with a fixed
appliance.
The major reason for loss of transplanted autogenous third molars is
A. infection.
B. immune rejection.
C. traumatic occlusion.
D. inadequate root length.
An "emergency" occlusal separator should be constructed for patients who has which of the
following conditions?
A. Myositis
B. Hemarthrosis
C. Capsular fibrosis
D. Degenerative arthritis
E. Unilateral condylar hyperplasia
To obtain the best long-term result, which of the following procedures for genioplasty is
recommended?
A. Silastic onlay implant
B. Proplast chin implant
C. Autogenous onlay bone graft

D. Pedicled horizontal sliding osteotomy


E. Allogeneic freeze-dried onlay bone graft
The most likely reason for a sudden temperature elevation six days after surgery is
A. S.B.E.
B. atelectasis.
C. wound infection.
D. urinary tract infection.
Which of the following groups of cells are significantly phagocytic?
A. Neutrophil and basophil
B. Basophil and eosinophil
C. Neutrophil and histiocyte
D. Neutrophil and lymphocyte
E. Plasma cell and lymphocyte
Which of the following is the least likely use of arthroscopy of the TMJ?
A. Diagnosis of disk perforations
B. Lysis of adhesions
C. Disk manipulation
D. Therapeutic lavage
E. Repair of disk perforations
Based on the Jones criteria, each of the following is a major manifestation of rheumatic fever
EXCEPT one. Which one is this EXCEPTION?
A. Fever
B. Chorea
C. Carditis

D. Polyarthritis
E. Erythema marginatum
Which of the following is usually NOT used for IV conscious sedation in the dental office
because of its long duration of clinically significant sedative effect?
A. Diazepam
B. Lorazepam
C. Midazolam
D. Hydroxyzine
E. Propofol
Each of the following is an advantage of midazolam over diazepam EXCEPT one. Which one is
this EXCEPTION?
A. Less incident of thrombophlebitis
B. Shorter elimination half-life
C. No significant active metabolites
D. Less potential for respiratory depression
E. More rapid and predictable onset of action when given intramuscularly
Which of the following represents the primary symptom to demonstrate that midazolam
(Versed) has been injected intra- arterially rather than intravenously?
A. Severe cardiac arrhythmia
B. Severe pain in the extremity
C. Rapid decrease in blood pressure
D. Rapid decrease in respiration rate
E. Exaggerated sedation due to the more direct effect of the drug
Which of the following clinical findings is generally NOT found in advanced osteoarthritis of the
temporomandibular joint?

A. Crepitus
B. Antegonial notching
C. Flattening of articular surface
D. Tendency toward apertognathia
E. Tendency toward prognathism
The majority of patients with "temporomandibular joint pain/dysfunction syndrome" have
A. normal dentitions.
B. no obvious TMJ pathology on panorex.
C. physical findings of rheumatoid arthritis in other joints.
D. evidence of destruction of cortical bone of the condylar head on the affected side.
A 46-year-old male presents for multiple extractions. For the last 10 years, he has been taking 10
mg of prednisone daily for colitis. After consulting with the patient's physician, the practitioner
should
A. discontinue the patient's steroid therapy preoperatively.
B. halve the patient's dose of prednisone to 5 mg on the day of surgery.
C. switch the patient to another steroid for the perioperative period.
D. ask the patient to continue taking the prednisone and consider temporarily increasing the
dose.
E. send the patient for serum prednisone levels.
What percentage of liquid is in an oxygen "E" cylinder when it is half full?
A. Less than 1%
B. .25
C. .5
D. .95

The nitrous oxide/oxygen sedation machine is set at 3 liters/min oxygen and 4 liters/min nitrous
oxide. What is the concentration of oxygen administered?
A. 33 percent
B. 43 percent
C. 53 percent
D. 63 percent
E. 73 percent
When both nitrous oxide and oxygen tanks are used to one-half of their capacities, what are their
gas pressures in psi (The starting gas pressures are 02 [2,000]; N20 [750])?
A. O2 = 1000; N2O = 325
B. O2 = 2000; N2O = 750
C. O2 = 1000; N2O = 750
D. O2 = 2000; N2O = 325
Each of the following describes nitrous oxide EXCEPT one. Which one is this EXCEPTION?
A. It is widely used by hospital anesthesiologists.
B. It is essentially not metabolized.
C. It minimally alters cardiac output.
D. It is heavier than air.
E. It is explosive.
Which of the following describes the titration of diazepam to Verrill's sign for IV conscious
sedation?
A. It is recommended as an end-point.
B. It is recommended only when supplemental 02 is used.
C. It is usually not attainable with diazepam alone.
D. It is not recommended since it can indicate a too-deeply sedated patient.

E. It is not recommended since few patients are adequately sedated at that level.
The only joint structure that exerts posterior traction on the articular disc is the
A. inferior retrodiscal lamina.
B. superior retrodiscal lamina.
C. inferior head of the lateral pterygoid.
D. superior head of the lateral pterygoid.
When surgically entering the superior joint space, one must incise through the
A. discal ligament only.
B. capsular ligament only.
C. capsular and discal ligaments.
D. capsular and retrodiscal lamina.
If there is insufficient tissue from the oral mucosa to close an alveolar cleft, the MOST common
method for obtaining soft tissue coverage is by using
A. dermis.
B. fascia lata.
C. a tongue flap.
D. Teflon-proplast.
E. freeze-dried dura.
The displacement of the proximal segment in an unfavorable angle fracture of the mandible
would be caused by which of the following muscles?
A. Digastric
B. Geniohyoid
C. Medial pterygoid
D. Lateral pterygoid

Inadvertent crush injuries to the lip are caused by which of the following parts of the forcep?
A. Beak
B. Hinge
C. Shank
D. Blade
E. Handle
Postextraction alveolar osteitis most commonly develops in how many days?
A. 1
B. 3
C. 5
D. 7
E. 10
The dentist is preparing a primary mandibular first molar for a stainless steel crown. Which of
the following tooth surfaces will require the least amount of reduction?
A. Buccal
B. Mesial
C. Lingual
D. Occlusal
A child has a very sore ulceration of the lower lip. There is no history of obvious trauma. The
ulceration appeared several hours after the patient received dental treatment. Which of the
following represents the most likely diagnosis?
A. Herpes labialis
B. Postanesthetic lip bite
C. Chemical burn from aspirin
D. Aphthous ulcer secondary to stress

E. Allergic stomatitis to the rubber dam


Which of the following should be done when a patient presents with a low, attached maxillary
frenum, accompanied by a 3 mm diastema in the early transitional dentition?
A. Perform a frenectomy when first observed
B. Observe until after eruption of permanent canines
C. Render treatment before complete eruption of maxillary incisors
D. Instruct patient to wear an elastic around teeth to close diastema
Which of the following should be included to ensure the best prognosis in the management of
localized juvenile periodontitis?
A. Systemic antibiotic therapy
B. Chlorhexidine mouthrinses
C. High doses of vitamin C
D. Free gingival grafts
E. Peroxide rinses
Modifying fear by familiarization is a successful method to use with children who
A. are handicapped.
B. have a behavior problem.
C. mirror their parents' fear of the dentist.
D. have had previous traumatic medical experience.
Which of the following diagnostic criteria is the least reliable in assessing the pulp status in the
primary dentition?
A. Swelling
B. Pulp testing
C. Spontaneous pain
D. Internal resorption

Daily cleaning of the root surface by the patient has been shown to
A. cause root resorption.
B. cause root sensitivity.
C. stimulate the epithelial attachment.
D. allow remineralization of the root surface.
Which of the following is the least effective in removing crevicular plaque?
A. Toothpick
B. Dental floss
C. A soft, nylon toothbrush
D. A water-irrigating device
Inflammation from periodontal disease usually extends to the bone marrow following
A. the nutrient canals.
B. primary trauma from occlusion.
C. secondary trauma from occlusion.
D. the course of the vascular channels.
E. the course of the periodontal ligament.
To prove its clinical effectiveness, an antimicrobial agent must demonstrate that it
A. reduces plaque mass.
B. helps to reduce disease.
C. specifically kills aerobic bacteria.
D. specifically kills anaerobic bacteria.
Following flap surgery, new junctional epithelium can form on either cementum or dentin.
Junctional epithelium is reestablished as early as one week.
A. Both statements are TRUE.

B. Both statements are FALSE.


C. The first statement is TRUE, the second is FALSE.
D. The first statement is FALSE, the second is TRUE.
Each of the following describes the experimental gingivitis model EXCEPT one. Which one is
this EXCEPTION?
A. It demonstrates the relationship between plaque formation and gingivitis.
B. It demonstrates that the bacterial ecology changes as plaque accumulates.
C. It demonstrates that gingivitis is a reversible disease.
D. It proves that gingivitis progresses to periodontitis.
E. It supports the non-specific plaque hypothesis.
A 35-year-old patient presents with swelling facial to a mandibular premolar. There is no severe
pain, only mild discomfort. The involved teeth are vital and there are no periodontal pockets.
Radiograph reveals a round radiolucency in the middle third of the root. Which of the following
represents the most likely diagnosis?
A. Gingival abscess
B. Lateral periodontal cyst
C. Osteogenic sarcoma
D. Periodontal abscess
E. Root resorption
The facial surface of the posterior mandible often presents limiting factors to proper treatment of
periodontal defects. Which of the following is primary among these factors?
A. Genial tubercles
B. Mentalis muscle
C. External oblique ridge
D. Internal pterygoid muscle

Which of the following landmarks will best guide the most esthetic location of the maxillary
anterior artificial teeth on a complete denture?
A. Incisive papilla
B. Midline of the nose
C. Median palatal suture
D. Anterior labial frenum
E. Anterior edentulous ridge
Which of the following describes the laterotrusive movement (Bennett angle)?
A. It is the angle that is formed by the non-working condyle and the sagittal plane during
lateral movements.
B. It is the angle that is formed by the condyle and the horizontal plane during protrusive
movements.
C. It is the difference in condylar inclination between protrusive and lateral movements.
D. It is the difference between the condylar and incisal inclinations.
In selecting an impression tray for a preliminary maxillary impression, how far should the tray
extend posteriorly?
A. To the tuberosity
B. To the fovea palatine
C. Beyond the vibrating line
D. To the pterygomaxillary notches
Why is it advisable to dispense the liquid component of cement immediately before mixing?
A. To avoid absorption of moisture from the air
B. To avoid spreading over a large area of the slab
C. To allow tempering of the powder by the mixing slab
D. To reduce the temperature influence of the mixing slab

E. To prevent evaporation of the volatile components


Which of the following groups of muscles are influential in molding the lingual border of the
final mandibular impression for a complete denture?
A. Hyoglossus, mylohyoid, medial pterygoid, digastric
B. Mylohyoid, styloglossus, medial pterygoid, depressor anguloris
C. Palatoglossus, superior pharyngeal constrictor, mylohyoid, genioglossus
D. Genioglossus, mylohyoid, middle pharyngeal constrictor, posterior belly of the digastric
The long axis of a mandibular second molar, Tooth #18, is mesially tipped 30 degrees to the
plane of occlusion. A 3-unit fixed partial denture from Tooth #18 to Tooth #20 is treatment
planned for this 70-year-old patient. Which of the following complications during preparation of
Tooth #18 would most likely affect the restoration's long-term prognosis?
A. Short distal axial wall
B. Irreversible pulpitis
C. Latent sensitivity
D. Pulp exposure
To obtain the best results when using an elastomeric impression material, the dentist must ensure
that the prepared tooth is
A. very dry.
B. free of surface moisture.
C. covered thinly with a water soluble lubricant.
D. covered with a surface tension-reducing agent and air dried.
E. lubricated with a petroleum-free gel exhibiting minimal film thickness and low viscosity.
A patient presents with a restricted floor of the mouth, only 6 mandibular anterior teeth, and
diastema between several teeth. Which of the following major connectors is appropriate for this
patient?
A. A lingual bar
B. A lingual plate

C. A lingual bar with a Kennedy bar


D. A lingual plate with interruptions (spaces) in the plate at the diastemas
Patients who have natural dentitions generate the greatest amount of occlusal force during
A. swallowing.
B. mastication.
C. centric relation.
D. parafunctional movements.
Over a period of years, which of the following is the most likely reason for the breakage of a
maxillary denture along the midline?
A. Alveolar resorption
B. Porosity of the denture base material
C. Over-relief of the incisive papilla
D. Inadequate extension of the posterior palatal seal
Proximal grooves for a partial veneer crown preparation for a maxillary central incisor are placed
parallel to the
A. long axis of the tooth.
B. long axis of the clinical crown.
C. incisal one-third of the facial surface.
D. incisal two-thirds of the facial surface.
Which of the following is a bactericidal antibiotic used specifically in the treatment of infections
caused by Pseudomonas species and indole-positive Proteus species?
A. Penicillin V
B. Tetracycline
C. Dicloxacillin
D. Carbenicillin

The ratio of LD50 to ED50 provides information as to the drug's


A. safety.
B. efficacy.
C. potency.
D. allergenicity.
E. therapeutic application.
A patient's early recovery from an ultrashort-acting barbiturate is related primarily to
A. redistribution.
B. breakdown in the liver.
C. excretion in the urine.
D. breakdown in the blood.
E. binding to plasma proteins.
Excretion of an acidic drug will be enhanced if the patient is given which of the following?
A. Mannitol
B. Aldosterone
C. Sodium chloride
D. Ammonium chloride
E. Sodium bicarbonate
Which of the following drugs is administered orally to treat vaginal candidiasis?
A. Fluconazole (Diflucan)
B. Griseofulvin (Grifulvin)
C. Clotrimazole (Mycelex Troche)
D. Miconazole (Monistat)

E. Nystatin (Mycostatin)
Respiratory difficulty due to blockade of the neuromuscular junction can be produced by which
of the following?
A. Penicillin
B. Sulfonamide
C. Streptomycin
D. Cephalosporin
E. Chloramphenicol
Three carpules (2 ml carpules, 40 mg/ml) of local anesthetic X are required to obtain adequate
local anesthesia. To obtain the same degree of anesthesia with local anesthetic Y, five carpules (2
ml carpules, 40 mg/ml) are required. If no other information about the two drugs is available,
then it is accurate to say that drug X
A. 0
B. is less potent than drug Y.
C. is more efficacious than Y.
D. is less efficacious than drug Y.
E. X&Y are = in potency & efficacy.
A patient who has Parkinson's disease is being treated with levodopa. Which of the following
characterizes this drug's central mechanism of action?
A. It replenishes a deficiency of dopamine.
B. It increases concentrations of norepinephrine.
C. It stimulates specific L-dopa receptors.
D. It acts through a direct serotonergic action.
Tardive dyskinesia is a neurological side effect of which of the following classes of drugs?
A. Alcohols
B. Tricyclic antidepressants

C. Barbiturate antiepileptics
D. Phenothiazine antipsychotics
E. Monoamine oxidase inhibitors
Which of the following penicillin drugs cannot be given orally?
A. Ampicillin
B. Cloxacillin
C. Methicillin
D. Penicillin G
E. Penicillin VK
Which of the following describes ciprofloxacin (Cipro)?
A. Inhibits cell wall synthesis.
B. Effective against Pseudomonas aeruginosa.
C. Effective only against anaerobic bacteria.
D. An antibiotic-of-choice for treating otitis media in young children.
E. Effective against oral anaerobes.
Which of the following drugs is an "irreversible" cholinesterase inhibitor?
A. DFP (Isoofluorophate)
B. Pilocarpine (pilocar)
C. Neostigmine (Prostigmine)
D. Physostigmine (Antilirum)
DFP and organophosphate insecticides produce their effects by
A. blocking nicotinic receptors.
B. blocking muscarinic receptors.

C. competitively inhibiting cholinesterases.


D. non-competitively inhibiting cholinesterases.
Which of the following is recommended for treating the pain of tic douloureux (trigeminal
neuralgia)?
A. Oxycodone
B. Ibuprofen
C. Carbamazepine
D. Hydrocortisone
E. Acetylsalicylic acid
Cycloplegia can be produced by
A. ephedrine.
B. pilocarpine.
C. amphetamine.
D. acetylcholine.
E. homatropine bromide.
Which of the following is an irreversible cholinesterase inhibitor?
A. Edrophonium
B. Physostigmine
C. Neostigmine
D. Pilocarpine
E. Diisopropylfluorophosphate
In a clinical pharmacologic comparison of atropine and its quatemary ammonium derivative
methylatropine nitrate, one might expect to observe that
A. atropine is more potent than methylatropine nitrate after oral administration.

B. methylatropine nitrate is more potent and effective than atropine in the treatment of
Parkinson's disease.
C. the ratio of the effective oral dose to the effective parenteral dose is higher for atropine
than for methylatropine nitrate.
D. in doses equieffective in affecting the salivary glands, drowsiness and clouding of
consciousness is a more prominent effect with methylatropine nitrate than with atropine.
Among cholinergic agents, a quatemary ammonium group in the structure of the drug appears
necessary for which of the following actions?
A. Nicotinic anticholinesterase
B. Direct nicotinic stimulation
C. Muscarinic anticholinesterase
D. Direct muscarinic stimulation
E. Central nervous system activity
Neostigmine is different from physostigmine because neostigmine
A. is not a clinically useful drug.
B. is well absorbed from the gastrointestinal tract.
C. has an additional direct effect at the neuromuscular junction.
D. is a reversible cholinesterase inhibitor while physostigmine is an irreversible one.
E. has the specificity to selectively block cholinesterase at the neuromuscular junction.
Which of the following statements is true regarding the directly acting cholinomimetics?
A. Methacholine has more prominent cardiovascular action than carbachol
B. Carbachol and bethanechol are almost completely resistant to action of ACHE
C. Bethanechol has some degree of selectively for gastrointestinal and urinary bladder
smooth muscle
D. All of these
The relationship of methacholine to acetylcholine is as

A. a vehicle.
B. a congener.
C. an isomer.
D. an adjuvant.
E. an antagonist.
The 1997 American Heart Association recommendations for prevention of bacterial endocarditis
includes which of the following dental procedures for antibiotic coverage?
A. Placement of rubber dams
B. Orthodontic appliance adjustment
C. Local anesthetic injections (nonintraligamentary)
D. Subgingival placement of antibiotic fibers or strips
E. Restorative dentistry with or without retraction cord
Methylparaben preservative most likely shows cross-sensitivity with which of the following
anesthetics?
A. Xylidides
B. Ortho-tuluidide derivatives
C. Para-aminobenzoic acid esters
D. Meta-aminobutyric acid esters
Antibiotics are obtained from growth of
A. fungi and viruses.
B. bacteria and fungi.
C. tissue cell cultures.
D. viruses and mycoplasmas.
E. mycoplasmas and rickettsiae.

Which of the following drugs is excreted primarily by renal tubular secretion?


A. Benzylpenicillin
B. Streptomycin
C. Tetracycline
D. Bacitracin
E. Polymyxin
Nalidixic acid is best described as
A. an antiviral agent.
B. an antifungal agent.
C. an antityphoid agent.
D. an antituberculous agent.
E. a urinary tract antiseptic.
Which of the following drugs may cause renal lithiasis (kidney stones) resulting from its limited
solubility in water?
A. Streptomycin
B. Erythromycin
C. Acetaminophen
D. Deteriorated tetracyclines
E. Sulfamethoxazole (a constituent of Bactrim DS)
Which of the following combinations is appropriate for treatment of uncomplicated respiratory
tuberculosis?
A. Streptomycin and chloramphenicol, because both are lipid- soluble.
B. Isoniazid (INH) and rifampin, because they have different modes of action.
C. Ethambutol and isoniazid, because both drugs inhibit bacterial folic acid synthesis.

D. Griseofulvin and streptomycin, because both drugs accumulate on bacterial 50S


ribosomes.
E. None of these
Which of the following is effective for treatment of a penicillinase-producing systemic
staphylococcal infection when taken orally?
A. Methicillin
B. Penicillin V
C. Streptomycin
D. Moxalactam
E. Oxacillin
Which of the following is a "third generation" cephalosporin?
A. Cephamandole
B. Moxalactam
C. Amikacin
D. Cephalexin
E. Clavulanic acid
Which of the following is most often associated with sensitivity of the skin to sun's rays?
A. Demeclocycline
B. Penicillin V
C. Erythromycin
D. Lincomycin
E. Nystatin
Which of the following tetracyclines is most slowly excreted?
A. Doxycycline

B. Tetracycline
C. Oxytetracycline
D. Chlortetracycline
Each of the following is an endogenous opioid peptide EXCEPT one. Which one is this
EXCEPTION?
A. Endorphin
B. Dynorphin
C. Bradykinin
D. Enkephalin
Which of the following penicillins would be used to treat a Pseudomonas infection?
A. Nafcillin (Unipen)
B. Amoxicillin (Amoxil)
C. Benzathine penicillin (Bicillin)
D. Phenoxymethyl penicillin (Pen-Vee K)
E. Ticarcillin (Thar)
3.6 ml solution of 4% prilocaine contains how many mg?
A. 3.6
B. 36
C. 72
D. 144
E. 360
Which of the following statements best describes the action of local anesthetics in inflamed
tissue? Local anesthetics
A. diffuse or penetrate more rapidly to the nerve due to their ionic charge.

B. diffuse or penetrate more rapidly to the nerve because they are predominantly nonionized.
C. are less efficacious because of the decreased pH of extracellular fluid.
D. are quickly inactivated by inflammatory mediators.
E. generally have a longer duration of action.
A 35-year-old patient who has an acute suppurative pulpitis is scheduled for tooth extraction.
Due to this patient's history of rheumatic heart disease, the practitioner administered penicillin
chemoprophylaxis. Fifteen minutes later, this practitioner notices that the periorbital tissue and
the lips of the patient are edematous. These findings are consistent with a diagnosis of
A. hematoma formation.
B. angioneurotic edema.
C. congestive heart failure.
D. an acute anaphylactoid reaction.
Postural hypotension is a common complaint of patients who take antihypertensive agents
because many of these agents interfere with the
A. sympathetic control of vascular reflexes.
B. release of acetylcholine in the ganglia.
C. epinephrine release from the adrenal medulla.
D. parasympathetic control of vascular resistance.
E. neuromuscular transmission in skeletal muscles.
Which of the following tetracyclines with a long biological half- life allows once-per-day
dosage?
A. Doxycycline
B. Oxytetracycline
C. Chlortetracycline
D. Demethylchlortetracycline

Which of the following is the preferred pharmalogic treatment for a xerostomia?


A. 5 mg Pilocarpine
B. 1 mg Epinephrine
C. 5 mg Levonordefrin
D. 1 mg Malathion
Which of the following penicillins would be used to treat a Pseudomonas infection?
A. Ticarcillin (Ticar )
B. Ampicillin (Omnipen )
C. Cloxacillin (Tegopen )
D. Phenoxymethyl penicillin (Pen-Vee K)
Drug interaction resulting in serious adverse cardiovascular events, including death, might occur
between erythromycin and which of the following antihistamine drugs?
A. Terfenadine (Seldane)
B. Promethazine (Phenergan)
C. Hydroxyzine (Atarax)
D. Diphenhydramine (Benadryl)
E. Chlorpheniramine (Chlortrimenton)
Hypersensitivity reactions to the penicillin derivatives include which of the following classical
types?
A. Type I only
B. Types I, II, and IV
C. Type IV only
D. Types I, II, Ill, and IV
Which of the following agents is used for HIV infections?

A. Amantadine (Symmetrel)
B. Acyclovir (Zovirax)
C. Zidovudine (Retrovir)
D. Ribavirin (Virazole)
E. Isoniazid (Nydrazid)
Which of the following drugs induced the first dramatic remission in acute leukemia in children?
A. Mechlorethamine
B. Carmustine
C. Aminopterin
D. Streptozocin
E. Mercaptopurine
Which of these opioid analgesics is associated with a serious life threatening drug interaction
when administered with an MAO inhibitor?
A. meperidine
B. morphine
C. fentanyl
D. propoxyphene
E. codeine
Which of the following groups of drugs is contraindicated for patients who have glaucoma?
A. Adrenergic
B. Cholinergic
C. Anticholinergic
D. Adrenergic blocking
Cholinomimetic drugs can produce which of the following effects?

A. Increased salivation
B. Peripheral vasoconstriction
C. Decreased gastric secretions
D. Decreased gastrointestinal peristalsis
E. Increased force of myocardial contraction
Each of the following is a pharmacologic effect of phenothiazines EXCEPT one. Which one is
this EXCEPTION?
A. Sedation
B. An antiemetic effect
C. Alpha-adrenergic blockade
D. Potentiation of the action of narcotics
E. An anticonvulsant
The extrapyramidal syndrome seen with the antipsychotic agents is due to their action on the
A. cerebellum.
B. brain stem.
C. hypothalamus.
D. basal ganglia.
E. cerebral cortex.
Which of the following compounds is used as an antiviral agent?
A. Amantadine (Symmetrel)
B. Novobiocin
C. Miconazole (Monistat)
D. Amphotericin B
Which pair of anesthetics is most likely to show cross-allergy?

A. Lidocaine - mepivacaine
B. Prilocaine - tetracaine
C. Procaine - mepivacaine
D. Procaine - lidocaine
E. Lidocaine benzocaine

Each of the following is an electromagnetic radiation EXCEPT one. Which one is the
EXCEPTION?
A. Radar
B. X rays
C. Alpha rays
D. Gamma rays
E. Visible light

Question 10 of 100
Which of the following physical signs indicates severe CNS oxygen deprivation?
A. Dilated pupils with increased light reflex
B. Pinpoint pupils with increased light reflex
C. Dilated pupils with an absence of light reflex
D. Pinpoint pupils with an absence of light reflex
Subconjunctival hemorrhage is most commonly found in which of the following fractures?
A. Nasal
B. LeFort I
C. Frontal sinus

D. Zygomatic arch
E. Zygomaticomaxillary complex

Question 16 of 100
The use of study casts in orthognathic surgery does each of the following EXCEPT one. Which
one is the EXCEPTION?
A. Constructs splints
B. Performs model surgery
C. Determines the postoperative occlusion
D. Identifies the type of skeletal deformity
E. Aids in explaining the surgical procedure to the patient

Question 18 of 100
Acute pyogenic bacterial infections produce
A. leukopenia.
B. lymphopenia.
C. neutropenia.
D. leukocytosis.
E. lymphocytosis.
Which of the following is indicated to correct a condition of atropine poisoning evidenced by
rapid heart rate, dry mouth and gastrointestinal inactivity?
A. Nicotine
B. Homatropine
C. Epinephrine
D. Pilocarpine

E. Physostigmine
Which of the following should be performed first for a collapsed or unconscious victim of illness
or accident?
A. Open the airway
B. Establish unresponsiveness
C. Establish pulselessness
D. Examine the victim for bleeding and fractures
A patient presents with facial lacerations that have irregular or devitalized wound edges. In
excising the wound elliptically, a practitioner takes into account each of the following EXCEPT
one. Which is the EXCEPTION?
A. Excision is perpendicular to the natural skin tension lines.
B. Length to width ratio of the excision is at least 3 to 1
C. Excision is done as conservatively as possible.
D. The wound is undermined prior to closure.
Which of the following represents the major route of excretion of penicillin V?
A. Secretion in the bile and into the feces
B. Excretion in the urine as inactive metabolites
C. Total metabolism by the liver and excretion in the feces
D. Secretion not metabolized in the urine
Which of the following systemic conditions lowers resistance, impairs healing, and indicates
early use of antibiotics for infections?
A. Vitamin C deficiency
B. Diabetes mellitus
C. Polycythemia vera
D. Systemic lupus erythematosus

What space management treatment should be planned for a child age 4, missing both primary
mandibular first molars with no primate space present?
A. Lingual arch
B. Distal shoe(s)
C. Nance holding arch
D. Band-and-loop space maintainer(s)
E. Observation; no space maintainer necessary
"PRIMATE SPACE" is more specific:
In the mandibular arch, the primate space is between the canine and the 1st molar
(or 1st premolar in adults).
Whereas, in the maxillary arch, it is between the lateral incisors and canines.

An unconscious patient is suspected of having an obstructed airway. How should this patient be
managed?
A. Protrude the tongue, clear the pharynx, extend the neck, and protrude the mandible
B. Extend the neck, clear the pharynx, protrude the tongue, and protrude the mandible
C. Extend the neck, protrude the mandible, protrude the tongue, and clear the pharynx
D. Clear the pharynx, extend the neck, protrude the mandible, and protrude the tongue
Which of the following is the least helpful in determining the anatomical integrity of the
temporomandibular joint?
A. Arthrography
B. Arthroscopy
C. Panoramic films
D. Computerized tomography
E. Magnetic resonance imaging

Question 32 of 100

A freshly condensed Class II amalgam restoration has a deficient margin at the proximogingival
cavosurface angle. This might have been caused by which of the following?
A. Overtightening the matrix band
B. Neglecting to wedge the matrix band
C. Neglecting to contour the matrix band
D. Using too large an initial increment of amalgam
Each of the following measures can minimize fractures of the maxillary alveolar process
EXCEPT one. Which one is the EXCEPTION?
A. Use of controlled force when using forceps and elevators
B. Removal of buccal bone and/or sectioning of teeth
C. A thorough presurgical analysis and planning alterations in the surgical approach
D. Use of the maxillary pinch grasp to detect expansion of the alveolar bone
E. Use of forceps with beaks that grasp the roots more firmly
Ethyl alcohol is a good antidote for methanol poisoning because
A. it inhibits methanol metabolism.
B. it competes successfully with methanol for alcohol dehydrogenase.
C. it prevents formation of formaldehyde.
D. it prevents damage to the optic nerve.
E. all of the above.
Each of the following nonsedating antihistamines would be contraindicated in an individual
taking cimetidine for heartburn EXCEPT one. Which one is the EXCEPTION?
A. Astemizole (Hismanal)
B. Diphenhydramine (Benadryl)
C. Fexofenadine (Allegra)
D. Hydroxyzine (Vistaril)

E. Terfenadine (Seldane)
Which of the following is a local contraindication for the surgical removal of an impacted tooth?
A. A nonvital tooth
B. Fever of unknown origin
C. A history of bleeding disorder
D. An undiagnosed ulcer in the oral cavity
E. Pale gingiva with a confirmed history of anemia

Question 37 of 100
Which of the following properties increases when the intermediate chain of a local anesthetic
drug is lengthened?
A. Potency
B. Allergenicity
C. Solubility in water
D. Rate of biotransformation
Which of the following is NOT an advantage of chemical vapor sterilization?
A. Cycle time is short.
B. It will not char fabrics.
C. It will not cause corrosion.
D. There is no need for special ventilation.
E. Instruments are dry at the end of the cycle.
In the preauricular approach to the TMJ, which of the following anatomic structures, if damaged,
presents the potential for greatest morbidity?
A. Facial nerve
B. Parotid gland

C. Auriculotemporal nerve
D. Superficial temporal vein
E. Facial artery
Osteomyelitis of the mandible is most likely to develop because of
A. failure of pus to localize.
B. a resistant strain of bacteria.
C. low resistance of the patient.
D. lack of drainage from the infected area.
How does the growth of the cranial base generally relate in time to the growth of the jaws?
A. Follows
B. Precedes
C. Unrelated
D. Accompanies
E. Initially follows, then accompanies
Aplastic anemia is a serious toxic effect associated most frequently with which of the following
antibiotics?
A. Streptomycin
B. Tetracycline
C. Chloramphenicol
D. Chlortetracycline
The therapeutic index of a given drug indicates the relative
A. safety.
B. potency.
C. efficacy.

D. duration.
E. solubility.
Which of the following flap designs allows the best surgical access to the apical aspect of a tooth
root with the least reflection of soft tissue?
A. Envelope
B. Semilunar
C. Vertical release
D. Rotation pedicle
Principles for managing a localized alveolar osteitis after extraction of a mandibular first molar
include each of the following EXCEPT one. Which one is the EXCEPTION?
A. Flushing out debris with normal saline solution
B. Curetting the boney wall to promote bleeding
C. Placing a sedative dressing in the socket to protect exposed bone
D. Administering mild analgesic drugs as an adjunct to local treatment
When is the most appropriate time to permanently modify/alter the occlusion of an acute TMD
patient?
A. During the first visit, while the patient has the acute symptoms
B. Prior to initiating splint therapy, one week after the first visit
C. Only after the patient is symptom-free and has shown improvement during splint therapy
D. Never, until definitive orthodontic treatment has been started
You are doing some preprosthetic surgery on a patient in preparation for a new denture. Which of
the following statements about removal of her moderate-sized mandibular lingual tori is correct?
A. The tori should be removed using a large pear-shaped acrylic bur.
B. The tori should be removed by grooving the superior surface then shearing the torus off
with a mono-beveled chisel. The area is then smoothed with a bone file.

C. The tori should be removed by grooving the superior surface of the torus with a bur, then
shear the torus off with a bibeveled chisel.
D. The tori should be removed by inserting a rongeurs vertically under a lingual flap and
"snipping off' the tori.
Which of the following statements about the flap for the removal of a palatal torus is correct?
A. The most optimal flap uses a midline incision which courses from the papilla between
teeth #8 and 9 posteriorly to the junction of the hard and soft palates.
B. The most optimal flap is a reflection of the entire hard palate mucoperiosteum back to a
line between the 2 first molar teeth.
C. The most optimal flap uses a midpalatal incision that courses from the palatal aspect of
tooth #3 across to the palatal aspect of tooth #14
D. The most optimal flap is shaped like a "double-Y", with a midline incision and anterior
and posterior side arms extending bilaterally from the ends of the midline incision.
A properly executed posterior superior alveolar nerve block will anesthetize each of the
following structures EXCEPT one. Which one is the EXCEPTION?
A. Maxillary sinus membrane in the molar region
B. Second and third molar teeth and a portion of the first molar tooth
C. Buccal alveolar bone, soft tissue and periodontium in the posterior maxilla
D. Soft palate mucosa on the side of the injection
Which areas should be avoided for an IM injection before a child can walk?
A. Gluteal muscles
B. Ventrogluteal
C. Vastus lateralis
D. Deltoid area
Which of the following antibiotics has been implicated in the vast majority of cases of oral
contraceptive failure during antibiotic use?
A. Penicillin

B. Rifampin
C. Keflex
D. Erythromycin
E. Tetracycline
Which of the following is the most serious result of digoxin intoxication?
A. Bradycardia
B. Renal failure
C. Atrial tachycardia
D. Atrial fibrillation
E. Ventricular fibrillation
Each of the following represents an advantage of oral sedation EXCEPT one. Which one is the
EXCEPTION?
A. Need for specialized training and equipment
B. High incidence and severity of adverse reactions
C. Erratic and incomplete absorption of drugs from the GI tract
D. Short duration of action
Spontaneous gingival hemorrhage or acute stomatitis, observed in a pediatric patient under
treatment for absence seizures, indicates withdrawal of which of the following anticonvulsant
drugs?
A. Phenytoin
B. Ethosuximide
C. Carbamazepine
D. Phenobarbital
E. Valproic acid
Back pressure porosity is

A. the same as occluded gas porosity.


B. the result of using an oxidizing flame.
C. most likely when the wax pattern is positioned very close to the open end of the ring.
D. often evidenced by rounded margins on the casting.
E. the same as suck-back porosity.
Which of the following explains why a properly designed rest on the lingual surface of a canine
is preferred to a properly designed rest on the incisal surface?
A. The enamel is thicker on the lingual surface.
B. Less leverage is exerted against the tooth by the lingual rest.
C. The visibility of, as well as access to, the lingual surface is better.
D. The cingulum of the canine provides a natural surface for the recess.
The main metabolite of ingested isopropyl alcohol is
A. urea.
B. acetone.
C. methanol.
D. acetaldehyde.
E. formaldehyde.
With respect to temperature, the processing cycle of a denture is designed to
A. attain development of color.
B. prevent boiling of the monomer.
C. prevent breakdown of the initiator.
D. volatilize the hydroquinone inhibitor.
The most likely cause of failure in the pre-ceramic soldering technique is
A. under heating the parts to be joined.

B. overheating the parts to be joined.


C. using an incompatible solder.
D. using phosphate as the soldering investment.
E. preheating the soldering assembly.
Alcoholic euphoria results from
A. increased activity of the cerebrum.
B. increased activity of thalamic areas.
C. increased activity of limbic synapses.
D. decreased activity of medullary centers.
E. removal of the inhibitory effect of the cortex.
The portion of an artificial tooth that is found only in porcelain anterior teeth is
A. the pin.
B. the collar.
C. the finish line.
D. the diatoric.
E. None of above
The casting shrinkage of cobalt-chromium alloys is approximately
A. 1.25%.
B. 1.75%.
C. 2.2%.
D. 3.2%.
Which of the following is a disadvantage of glass ionomer cement?
A. Difficulty in mixing

B. Irritation of the pulp


C. Low bond strength to dentin
D. Moisture sensitivity during initial set
The base of the incision in the gingivectomy technique is located
A. in the alveolar mucosa.
B. at the mucogingival junction.
C. above the mucogingival junction.
D. coronal to the periodontal pocket.
E. at the level of the cementoenamel junction.
The most important anticoagulant effect of heparin is to interfere with the conversion of
A. PTA to PTC.
B. PTC to Factor VIII.
C. fibrinogen to fibrin.
D. prothrombin to thrombin.
E. proaccelerin to accelerin.
One can accelerate the setting time of zinc oxide-eugenol impression pastes by adding a small
amount of
A. water.
B. glycerin.
C. petrolatum.
D. plaster of paris
When determining the appropriate dose of systemic fluoride supplement for a child, it is most
important for the dentist to consider which of the following?
A. The fluoride content of the drinking water

B. The child's diet and caries activity


C. The child's age and the fluoride content of the drinking water
D. The child's weight and the fluoride content of the drinking water
The drug of choice to treat overdosage with tricyclic antidepressants is
A. atropine.
B. phenytoin.
C. physostigmine.
D. pentobarbital.
E. an amphetamine
Drug interaction resulting in serious adverse cardiovascular events, including death, might occur
between erythromycin and which of the following antihistamine drugs?
A. Terfenadine (Seldane)
B. Promethazine (Phenergan)
C. Hydroxyzine (Atarax)
D. Diphenhydramine (Benadryl)
E. Chlorpheniramine (Chlor-Trimeton)
A mercury spill in the laboratory or office is appropriately cleaned up by
A. removing the mercury, then spraying the area with BAL.
B. scrubbing the spill area with hot water and detergent.
C. sweeping up the mercury and disposing of it in a plastic bag.
D. aspirating the mercury into a wash bottle trap, then dusting the spill area with sulfur
powder.
A prescription includes each of the following parts EXCEPT one. Which one is the
EXCEPTION?
A. Superscription

B. Inscription
C. Subscription
D. Transcription
E. Conscription
Inhalation of amyl nitrite can result in each of the following EXCEPT one. Which is the
EXCEPTION?
A. Tachycardia
B. Coronary artery dilation
C. Peripheral arteriolar dilation
D. A decrease in arterial blood pressure
E. Increased motility of the small bowel
Which of the following drugs is commonly used in the treatment of congestive heart failure?
A. Phenytoin
B. Digitalis
C. Quinidine
D. Procainamide
E. Nitroglycerin
What percent of the blood alcohol level is the most likely to produce a lethal effect in 50 per cent
of the population?
A. 0.05%
B. 0.10%
C. 0.20%
D. 0.03%
E. 0.05%

Prolonged use of amyl nitrite may result in


A. aplastic anemia.
B. thrombocytopenia.
C. granulocytopenia.
D. methemoglobinemia.
E. hypoprothrombinemia.
Which of the following statements is true concerning anticholinesterase?
A. Agents such as DEP inhibit only plasma cholinesterase.
B. Organophosphates are readily absorbed through the skin.
C. Agents may cause paroxysmal supraventricular tachycardia.
D. Reactivators such as 2-PAM reactivate ACHE which has undergone "aging".
E. Alkaloid physostigmine has the greatest number of side effects unrelated to ACHE
inhibition.
Which of the following drug groups is currently the mainstay of treatment in depressive
psychoneurotic disease?
A. Amphetamines
B. Phenothiazines
C. Benzodiazepines
D. Monoamine oxidase inhibitors
E. Tricyclic (imipramine-like) antidepressants
When high gold content alloys are compared to base metal alloys, the base metal alloys exhibit
A. a higher melting point, increased specific gravity, and generally higher yield strength and
hardness.
B. a higher melting point, decreased specific gravity, and generally higher yield strength and
hardness.

C. a higher melting point, generally higher yield strength and hardness, and more consistent
bonding to porcelain.
D. increased specific gravity, generally lower yield strength, and more consistent bonding to
porcelain.
E. decreased specific gravity, generally higher yield strength and hardness, and more
consistent bonding to porcelain.
The central skeletal muscle relaxation produced by depressing the polysynaptic reflex arcs is
brought about by all of the following drugs EXCEPT
A. diazepam (Valium).
B. lorazepam (Ativan).
C. meprobamate (Equanil).
D. d-tubocurarine (Tubarine).
Which of the following statements best describes why L-dopa eventually becomes ineffective in
the treatment of Parkinson's disease?
A. L-dopa absorption slows with aging.
B. Dopa decarboxylase activity increases with age.
C. Dietary intake of pyridoxine speeds L-dopa metabolism.
D. Patients gradually develop tolerance because L-dopa induces liver enzymeactivity.
E. Neuronal cell loss in the substantia nigra is progressive and continuous over the course of
the disease.
The cardiac glycosides will reduce the concentration of which ion in an active heart muscle cell?
A. Sodium
B. Calcium
C. Chloride
D. Potassium
E. Magnesium

Which of the following drugs is excreted primarily by renal tubular secretion?


A. Benzylpenicillin
B. Streptomycin
C. Tetracycline
D. Bacitracin
E. Polymyxin
In pursuit of what the dentist believes is best for the patient, the dentist attempts to control
patient behavior. This is known as
A. autonomy.
B. competence.
C. maleficence.
D. paternalism.
When 50 mg. of chlorpromazine (Thorazine) is administered to a patient, on standing the
patient might experience a fall in blood pressure due to which of the following?
A. Anticholinergic action
B. Decrease in heart rate
C. Alpha-adrenergic blockade
D. Negative inotropic action
E. Stimulation of autonomic ganglia
Metabolism of a drug will usually result in conversion to each of the following EXCEPT one.
Which one is the EXCEPTION?
A. Inactive form
B. More active compound
C. Less active compound
D. More water-soluble compound

E. Less ionized compound


Which of the following sedatives is most likely to cause a dry mouth?
A. Buspirone (BuSpar)
B. Hydroxyzine (Vistaril)
C. Chloral hydrate (Noctec)
D. Phenobarbital (Luminal)
Which of the following erythromycins is both enteric coated and long acting?
A. ERYC
B. E.E.S.
C. Erythrocin
D. Ilosone
Each of the following is a characteristic of tramadol (Ultram) EXCEPT one. Which one is the
EXCEPTION?
A. Centrally acting analgesic
B. Structurally similar to morphine
C. Binds to the mu-opioid receptor
D. Biotransformed into a more active metabolite
E. Inhibits uptake of norepinephrine and serotonin
During a soldering procedure, flux serves to
A. provide an oxidizing environment in the area to be soldered.
B. displace gases and dissolve corrosion products.
C. remove any debris that may remain in the area to be soldered.
D. hold the solder in place during heating.
Excessive heating of the acrylic resin during processing should be avoided to prevent

A. exothermic heat buildup.


B. acrylic resin shrinkage.
C. evaporation of the monomer.
D. excessive acrylic resin expansion.
E. discoloration of the acrylic resin.
Following compression of acrylic into the denture flasks, placing the flasks into the processing
tanks at curing temperature is delayed to
A. assure complete flow of acrylic into the mold.
B. allow the monomer to permeate all polymer crystals.
C. establish an equalized and uniform pressure in the molds.
D. allows the flasks and the acrylic to reach a stable temperature.
Early stage periapical cemental dysplasia is best differentiated from chronic apical periodontitis
by
A. percussion.
B. subjective symptoms.
C. periodontal examination.
D. results of pulp testing.
E. radiographic appearance.
Which of the following represents the sequence in the replantation of mature, avulsed teeth?
A. Repositioning, splinting, and performing root canal therapy
B. Performing root-canal therapy, repositioning, and splinting
C. Performing canal debridement, performing calcium hydroxide therapy, repositioning,
splinting, and filling with gutta- percha
D. Repositioning, splinting, performing canal debridement, performing calcium hydroxide
therapy, and filling with gutta- percha

E. Repositioning and splinting only


If periapical radiolucency is present at the apex of a tooth with a middle third root fracture and
the apical canal space is non-negotiable, the required treatment would be
A. extraction because of the poor prognosis.
B. calcium hydroxide treatment of the coronal segment only.
C. apical surgery with a Super EBA reverse filling.
D. apical surgery with removal of the apical segment root canal treatment on the coronal
segment.
E. extraction with removal of the apical segment and replantation of the coronal segment.
A 7-year-old patient fractured the right central incisor three hours ago. A clinical examination
reveals a 2-mm exposure of a "bleeding pulp." The treatment-of-choice is
A. pulpectomy and apexification.
B. pulpotomy with calcium hydroxide.
C. direct pulp cap with calcium hydroxide.
D. one-appointment root canal treatment.
Which of the following outlines best describes the access cavity preparation on a mandibular
molar with four canals?
A. Oval
B. Trapezoidal
C. Triangular
D. Rectangular
Loss of the apical seat caused by overinstrumentation is best managed by
A. prescribing analgesics and antibiotics.
B. filling with a warm gutta-percha technique.
C. increasing the file size and decreasing the file length.

D. using a chelating agent and packing with calcium hydroxide.


E. filling with a gutta-percha cone that is larger than the apical perforation.
Which of the following is the most effective way to reduce injury to the pulp during a restorative
procedure?
A. Prepare dentin with slow-speed burs.
B. Use anesthetics without vasoconstrictors.
C. Minimize dehydration of the dentinal surface.
D. Keep the dentinal surface clean by frequent irrigation.
E. Use light, short duration contact with the bur.
Which of the following teeth have the most consistent canal morphology?
A. Mandibular incisors
B. Maxillary canines
C. Mandibular premolars
D. Maxillary premolars
E. Mandibular molars
Rigid splinting of an avulsed permanent incisor will most likely lead to
A. normal healing response.
B. calcific metamorphosis.
C. replacement resorption.
D. internal resorption.
E. root fracture.
Which of the following diagnoses represents an injury that causes the tooth to loosen but is not
displaced?
A. Concussion

B. Luxation
C. Subluxation
D. Extrusion
E. Intrusion
Waiting to allow intruded permanent central incisors to reerupt in an 8-year-old child will likely
lead to
A. a healing response and reeruption.
B. calcific metamorphosis.
C. internal resorption.
D. tooth discoloration.
E. ankylosis.
Which of the following factors is of greatest importance in selecting a restorative material for a
Class V cavity on the facial surface of a mandibular premolar?
A. Reaction of gingival tissue
B. Resistance to stress
C. Ease of manipulation
D. Esthetics
Before beginning tooth preparation, the dentist should visualize the outline form to
A. establish the convenience form.
B. establish the resistance and retention form.
C. prevent overcutting and overextension.
D. aid in the finish of enamel walls and margins.
The anterior maxillary incisors of a middle-aged patient can be given a younger appearance by
selective grinding of the enamel. The dentist performs this procedure by
A. flattening the incisal edges.

B. shortening the incisal edges.


C. rounding the incisal point angles.
D. moving the facial line angles proximally.
E. moving the facial height of curvature gingivally.
In radiography, a longer gray scale of contrast can be achieved by
A. decreasing the filtration.
B. increasing the kilovoltage.
C. increasing the milliamperage.
D. increasing the focal spot-skin distance.
As an x-ray tube operates, electrons carry energy from the cathode to the anode. Into which of
the following is most of this energy converted in the target?
A. Heat
B. X rays
C. Magnetism
D. Electricity
E. Visible light
The inferior concha is composed of all or part of which of the following bones?
A. Maxilla
B. Ethmoid
C. Sphenoid
D. Palatine
E. Turbinate
Each of the following could be ruled out for the lesion distal to the mandibular third molar
EXCEPT one. Which one is this EXCEPTION?

A. Cementoblastoma
B. Osteoma
C. Fibroma
D. Salivary gland inclusion defect (Stafne defect)
When making a bite-wing exposure of the posterior teeth, the recommended vertical angulation
may vary from
A. -10 degrees to -5 degrees
B. -5 degrees to 10 degrees
C. 0 degrees to +5 degrees
D. +5 degrees to +10 degrees
A panoramic radiograph of an asymptomatic adult discloses a well-defined 1.5 cm radiolucency
below the mandibular canal. The lesion is not palpable. Which of the following is the most likely
diagnosis?
A. Ameloblastoma
B. Traumatic bone cyst
C. Odontogenic keratocyst
D. Focal osteoporotic bone marrow defect
E. Salivary gland inclusion defect (Stafne)
Ectodermal dysplasia is characterized by each of the following EXCEPT one. Which one is this
EXCEPTION?
A. Hypodontia (oligodontia)
B. Blue sclera
C. Atrophic skin
D. Defective hair
E. Hypoplastic sweat glands

An 18-year-old man complains of tingling in his lower lip. An examination discloses a painless,
hard swelling of his mandibular premolar region. The patient first noticed this swelling 3 weeks
ago. Radiographs indicate a loss of cortex and a diffuse, radiating pattern of trabeculae in the
mass. Which of the following is the most likely diagnosis?
A. Leukemia
B. Dentigerous cyst
C. Ossifying fibroma
D. Osteosarcoma
E. Hyperparathyroidism
Malignant melanoma most commonly occurs in which of the following intraoral sites?
A. Buccal mucosa and vestibule
B. Lateral and ventral tongue mucosa
C. Soft palate and tonsillar pillar complex
D. Hard palate and maxillary gingiva
E. Floor of mouth and anterior lingual gingiva
A radiographic examination reveals a radiolucent area in the region of the mandibular left third
molar. The third molar is not present. The clinical differential diagnoses should include each of
the following EXCEPT one. Which one is this EXCEPTION?
A. Ameloblastoma
B. Residual cyst
C. Odontogenic keratocyst
D. Dentigerous cyst
E. Odontogenic myxoma
Radiographs of an asymptomatic, 20-year-old patient reveal a sharply outlined radiolucent lesion
in the mandibular first molar area. This 2 cm lesion scallops between the roots of vital teeth. The
most probable diagnosis is
A. radicular cyst.

B. dentigerous cyst.
C. traumatic bone cyst.
D. odontoma.
Examination reveals a soft, fluctuant, tender swelling in the midline of the hard palate. The teeth
test vital. Radiographs reveal a radiolucent area projected between the roots of the maxillary
central incisors. Which of the following cysts represents the most likely diagnosis?
A. Nasolabial
B. Median palatal
C. Periapical
D. Nasopalatine duct
E. Incisive papilla
Incorrect horizontal angulation of the x-ray tubehead during exposure of a molar bite-wing leads
to
A. elongation of teeth.
B. overlapping of interproximals
C. overexposure of film
D. foreshortening of roots.
E. foreshortening of crowns.

Question 89 of 100
Symptoms of pain and tenderness upon palpation of the temporomandibular joint are usually
associated with which of the following?
A. Impacted mandibular third molars
B. Flaccid paralysis of the painful side of the face
C. Flaccid paralysis of the non painful side of the face
D. Excitability of the second division of the fifth nerve

E. Deviation of the jaw to the painful side upon opening the mouth
A patient placed an aspirin directly in the mandibular facial sulcus. Shortly afterward, a wellcircumscribed white patch appeared on the mucosa. Which of the following is the most likely
diagnosis?
A. Keratosis
B. Hyperplasia
C. Atrophy
D. Necrosis
E. Hypertrophy
Increasing mA setting of an x-ray unit raises which of the following?
A. Photon speed
B. Photon wavelength
C. Number of photons generated
D. Voltage between anode and cathode
E. Effective energy of the resultant beam
Hypertrophy of the mandibular condyle in an adult can result in each of the following EXCEPT
one. Which one is this EXCEPTION?
A. Anterior open bite
B. Anterior crossbite
C. Ipsilateral posterior open bite
D. Unilateral Class III malocclusion
E. Asymmetric facial appearance
The hamular notch is important in complete denture construction BECAUSE it aids the dentist in
positioning the maxillary posterior teeth.
A. Both the statement and the reason are correct and related.

B. Both the statement and the reason are correct but NOT related.
C. The statement is correct, but the reason is NOT.
D. The statement is NOT correct, but the reason is accurate.
E. NEITHER the statement NOR the reason is correct.
The most common type of active tooth movement in the primary dentition is correction of which
type of problem?
A. Vertical
B. Sagittal
C. Transverse
D. Space regaining in the maxillary arch
E. Space regaining in the mandibular arch
A patient has a chronically tender, knife-edge mandibular residual ridge. In fabricating a
complete denture for this patient, a dentist should consider which of the following?
A. Maximal extension of the denture--to distribute forces of occlusion over a greater area
B. Minimal extension of the denture--to limit tenderness to a smaller area
C. Decreased occlusal vertical dimension--to decrease biting forces
D. A broad occlusal table--to provide a firmer contact in eccentric jaw relations
A patient presents with spontaneous necrotizing ulcers of the oral cavity. The WBC is 1,986,
while the differential CBC reveals lymphocytes--68%, monocytes--28%, polymorphonuclear
leukocytes--2%, eosinophils--l%, and basophils--l%. Which of the following diagnoses is the
MOST appropriate for this patient?
A. Leukemia
B. Infectious mononucleosis
C. Agranulocytosis
D. Recurrent aphthous ulcers

Individuals who are cognitively disabled (e.g. those who have Down syndrome) sometimes have
difficulty accepting dental care. In attempting to improve their cooperation, the dentist should do
which of the following as an initial therapy?
A. Administer sedative agents
B. Use graduated exposures to the dental setting
C. Use physical restraints
D. Refer the patient to a special care clinic
Oral hygiene improvement in the presence of systemic or nutritional deficiencies affects the
incidence of chronic inflammatory periodontal disease. It also affects the severity of chronic
inflammatory periodontal disease.
A. Both statements are TRUE.
B. Both statements are FALSE.
C. The first statement is TRUE, the second is FALSE.
D. The first statement is FALSE, the second is TRUE.
A patient who has which of the following conditions is most likely to have postoperative
bleeding after multiple extractions?
A. Angina
B. Diabetes
C. Cirrhosis
D. Rheumatic fever
E. Chronic bronchitis
Two adjacent cavities involving proximal contact can be prepared and restored with composite
resin at one appointment for each of the following reasons EXCEPT one. Which one is this
EXCEPTION?
A. Restoration of contact is enhanced.
B. Access to adjacent cavities is simplified.
C. Color matching is easier.

At birth, the palate is relatively flat; in adults, it is vault-shaped. By which of the following does
this change occur?
A. Bone resorption in the palatal vault
B. Growth of the maxillary sinuses
C. Deposition of the alveolar crestal bone
D. Bone deposition on the posterior wall of the maxillary tuberosity
A 50-year-old male presents with deep fibrotic pockets and angular bone loss. The dentist
reflects a flap, debrides the two and three-walled defects, and performs scaling and root planing.
Which procedure should the dentist do next?
A. Gingivectomy
B. Osseous grafts
C. Modified Widman flap
D. Apically repositioned flap
A patient who is on dicumarol therapy requires a tooth extraction. Which laboratory test is the
most valuable in evaluating this patient's surgical risk?
A. Clotting time
B. Bleeding time
C. Prothrombin time
D. Sedimentation rate
E. Complete blood cell count
In radiobiology, the "latent period" represents the period of time between
A. cell rest and cell mitosis.
B. the first and last dose in radiation therapy.
C. film exposure and image development.
D. radiation exposure and onset of symptoms.

A broad understanding of the development of human behavior requires knowledge of the basic
concepts of
A. maturation and learning.
B. dependence and independence.
C. gender and race.
D. generalization and facilitation.
What is the pH threshold level at which enamel demineralization occurs?
A. 6.5
B. 5.5
C. 4.5
Which of the following clinical lesions MOST often reveals histologic evidence of severe
dysplasia or carcinoma in situ?
A. Leukoedema
B. Leukoplakia
C. Lichen planus
D. Erythroplakia
E. White sponge nevus
Which of the following planes of space are used to classify malocclusion?
A. Horizontal, vertical, diagonal
B. Antero-posterior, sagittal, vertical
C. Antero-posterior, transverse, vertical
D. Antero-posterior, sagittal, coronal
Before adhesion can take place between a liquid and a solid, it is essential that the liquid surface
A. provide some mechanical interlocking with the solid.

B. exhibit a large contact angle with the solid.


C. enter into some form of chemical reaction with the solid.
D. exhibit a small contact angle with the solid.
Which of the following root surfaces is the most likely to be strip-perforated during canal
instrumentation of the mesial root of a mandibular first molar?
A. Facial
B. Lingual
C. Mesial
D. Distal
To increase the anterior mandibular vestibular depth, the practitioner places the mucosal graft
directly on the
A. mucosa.
B. periosteum.
C. muscle.
D. bone.
Which of the following generally describes children with defiant behavior?
A. They are usually around 3 years of age or younger.
B. They are also referred to as stubborn or spoiled.
C. They usually exhibit tremors in voice when speaking.
D. They exhibit quiet sobbing with profuse tears.
A patient has a mandibular molar with pulp necrosis, pain to percussion, and with no
periradicular lesions. The emergency treatment. for this tooth is which of the following?
A. Opening the tooth, performing apical trephination, temporizing, and checking the
occlusion
B. Prescribing analgesics and antibiotics and rescheduling the patient

C. Performing pulpotomy, temporizing, and checking the occlusion


D. Debriding the canals, temporizing, and checking the occlusion
E. Opening the tooth for drainage and leaving the tooth open
Local anesthetics aid in reducing the flow of saliva during operative procedures by
A. blocking the cholinergic nerve endings.
B. blocking innervation to major salivary glands.
C. blocking efferent parasympathetic nerve pathways.
D. reducing sensitivity and anxiety during tooth preparation.
In a complete maxillary denture, accurate adaptation of the border of the maxillary facial flange
affects which of the following the most?
A. Speech
B. Support
C. Stability
D. Esthetics
Fixing solution functions to
A. accelerate the film processing.
B. remove the atomic silver.
C. remove undeveloped silver salts.
D. reduce silver ions to metallic silver.
Which of the following patients should preferably be hospitalized for the oral surgery procedure
indicated?
A. A patient with a prosthetic cardiac valve, who requires a single tooth removed, and an
alveoloplasty procedure.
B. A severely retarded patient, who requires full mouth gingivectomy for treatment of
phenytoin-induced hyperplasia.

C. A patient on anticoagulant therapy whose present prothrombin time is 1.25 times the
control time, who requires removal of a single tooth.
D. A juvenile diabetic whose disease is controlled by daily insulin injections, who requires
removal of a chronically infected tooth.
A dentist uses an unsaturated, chemical sterilizer for sterilizing dental instruments. To effectively
kill all microorganisms and spores, this dentist must maintain a temperature of 132 degrees
Celsius at 20 - 40 psi for at LEAST how many minutes?
A. 10
B. 20
C. 30
D. 40
There are four basic stages in the pathogenesis of the periodontal lesion. In progressing order,
they are
A. initial, established, early, and advanced.
B. early, incipient, established, and advanced.
C. initial, early, established, and advanced.
D. incipient, early, established, and advanced.
Which of the following combinations of agents would be necessary to block the cardiovascular
effects produced by the injection of a sympathomimetic drug?
A. Atropine and prazosin
B. Atropine and propranolol
C. Prazosin and propranolol
D. Phenoxybenzamine and curare
E. Amphetamine and propranolol
A cleft lip occurs following the failure of permanent union between which of the following?
A. The palatine processes

B. The maxillary processes


C. The palatine process with the frontonasal process
D. The maxillary process with the palatine process
E. The maxillary process with the frontonasal process
The highest incidence of fibrous gingival enlargement (hyperplasia) is related to
A. puberty.
B. diabetes.
C. leukemia.
D. pregnancy.
E. medication.
A patient has two or more of her teeth joined only by cementum. This phenomenon is known as
A. fusion.
B. hypercementosis.
C. gemination.
D. dilaceration.
E. concrescence.
While the dentist is preparing a large carious lesion in Tooth #30 for a restoration, a pulp
exposure occurs. The patient angrily shouts at the dentist, "You incompetent 'creep'--you're
responsible for this problem!"- Of the following possible responses the dentist could make,
which one is the most emphatic?
A. Calm down, I can still restore your tooth adequately.
B. Not when I'm preparing a tooth with caries like you had.
C. I can see that you're very upset. You thought the tooth could be restored and now this
problem has occurred.
D. If you took care of your mouth the way you should, I wouldn't have been close to the
pulp.

E. I'm sorry this happened, but we must get on with the procedure.
Which of the following is the most serious toxic effect that is associated with overdose of
acetaminophen?
A. Hemorrhage
B. Renal necrosis
C. Hepatic necrosis
D. Gastric ulceration
E. Respiratory alkalosis
A patient needs relining of both maxillary complete denture and mandibular distal-extension
partial denture. The recommended procedure is to
A. make impressions of both simultaneously.
B. start first on the maxillary complete denture.
C. start first with the least stable prosthesis.
D. relate first the mandibular partial denture frame.
In which of the following do adenomatoid odontogenic tumors MOST often occur?
A. Anterior maxilla
B. Posterior maxilla
C. Anterior mandible
D. Posterior mandible
Duration of treatment regimens affect patient compliance. The severity of disease perceived by
the patient affects patient compliance.
A. Both statements are TRUE.
B. Both statements are FALSE.
C. The first statement is TRUE, the second is FALSE.
D. The first statement is FALSE, the second is TRUE

The Centers for Disease Control (CDC) recommends that sterilizing units be monitored at which
of the following intervals?
A. Daily
B. Weekly
C. Bi-weekly
D. Monthly
An evaluation of which of the following represents the most important aspect in shade selection
(for the restoration to match an existing dentition)?
A. Hue
B. Value
C. Chroma
D. Size
E. Shape
A fracture through the angle of the mandible can result in an upward displacement of the
proximal fragment. Which of the following groups of muscles produces this movement?
A. Digastric and geniohyoid
B. Masseter, digastric, and lateral pterygoid
C. Masseter, temporal, and lateral pterygoid
D. Masseter, temporal, and medial pterygoid
During an emergency dental visit, a moderately retarded 4-year-old child resists physically and
cries excessively. In this instance, the dentist should use which of the following?
A. Physical restraints
B. General anesthesia
C. The hand-over-mouth exercise
D. Voice control

A practitioner is restoring the mesio-occlusal marginal ridge of a maxillary left second molar. If
the marginal ridge is higher than the adjacent tooth, then it can create a problem in which of the
following excursions?
A. Protrusive
B. Retrusive
C. Working
D. Non-working
Lymphangioma is MOST closely related to
A. hemangioma.
B. angiosarcoma.
C. cystic hygroma.
D. hemangiopericytoma.
According to the American Heart Association, which of the following prophylactic antibiotic
regimens is recommended for a 20-kg child who has congenital heart disease?
A. 1.0 gram amoxicillin orally one hour before the dental procedure and 500 mg 4 times a
day for 2 days postoperatively
B. 1.0 gram penicillin V orally one hour before the dental procedure and 500 mg orally 6
hours later
C. 1.0 gram amoxicillin one hour before the dental procedure and 500 mg orally 6 hours
later
D. 3.0 grams amoxicillin orally one hour before the dental procedure and 1.5 grams orally 6
hours later
When compared therapeutically to penicillin G, penicillin V has a
A. slower renal excretion.
B. more reliable oral absorption.
C. broader antibacterial spectrum.
D. greater resistance to penicillinase.

E. lower potential for allergic reaction.


Which of the following teeth are usually affected by nursing-bottle caries?
A. Maxillary incisors
B. Mandibular molars
C. Mandibular incisors
D. Maxillary molars
An adult healthy patient has a marked indurated swelling. He has a temperature of 100 degrees
Fahrenheit and has been in considerable pain for 24 hours. Percussion of the maxillary right
central incisor causes discomfort as does palpation at its apex. Pulp vitality tests are negative for
this tooth, while adjacent control teeth test within normal limits. A radiograph reveals that the
maxillary right central incisor has a deep unbased restoration and a widened apical periodontal
ligament space. Which of the following is the best emergency treatment for this patient?
A. Extracting the maxillary right central incisor
B. Administering an antibiotic and analgesic and initiating root canal therapy when
symptoms subside
C. Debriding the root canal of the maxillary right central incisor and prescribing antibiotics
and analgesics
D. Incising and draining the swelling and prescribing antibiotics and analgesics
A decrease in which of the following causes an increase in radiographic density?
A. mA
B. kVp
C. Object-film distance
D. Source-object distance
A patient who has a Class III malocclusion presents for treatment. Each of the following is a
helpful diagnostic aid to distinguish between maxillary retrusion and mandibular protrusion
EXCEPT one. Which one is this EXCEPTION?
A. Photography
B. Cephalometry

C. Study models
D. Clinical evaluation
A dentist will use a buccal coil spring to regain space for a mandibular second premolar. Which
of the following is an undesirable side effect that is most commonly associated with this
procedure?
A. Pain
B. Gingival irritation
C. Severe mobility of the tooth
D. Tendency for the first molar to intrude
E. Tendency for the first premolar to rotate
Tooth bleaching affects a color change in which of the following tooth parts?
A. Dentin only
B. All of the enamel
C. Both dentin and enamel
D. Only the surface of enamel
In primary teeth, which, of the following most often causes calcium hydroxide pulpotomies to
fail?
A. Pulp calcification
B. Periapical pathosis
C. Internal resorption
D. Formation of dentin bridges
Each of the following diseases has been associated with the Epstein-Barr virus EXCEPT one.
Which one is this EXCEPTION?
A. Burkitt's lymphoma
B. Oral hairy leukoplakia

C. Infectious mononucleosis
D. Adenoid cystic carcinoma
In a 12-year-old, an ANB angle of 2 degrees usually signifies that the
A. relationship of the maxilla to the mandible is favorable.
B. relationship of the mandible to the cranium is favorable.
C. cranial growth is poor.
D. incisors are upright.
Which of the following mechanical properties limits the amount of adjustment of a base metal
removable clasp arm?
A. Hardness
B. Stiffness
C. Elongation
D. Tensile strength
Each of the following is a common side effect of prolonged tetracycline therapy EXCEPT one.
Which one is this EXCEPTION?
A. Diarrhea
B. Superinfection
C. Photosensitivity
D. Visual disturbance
E. Discoloration of newly forming teeth
How do the surface characteristics of a restoration affect its perceived form?
A. A surface smoother than normal will give the impression of a larger size.
B. Increasing the value of the restoration makes the tooth appear smaller.
C. Horizontal highlights give an illusion of increased length.

D. Vertical highlights give an illusion of increased width.


Bilateral enlargement of the parotid glands characterizes each of the following conditions
EXCEPT one. Which one is the EXCEPTION?
A. Malnutrition
B. Sialolithiasis
C. Sjogren's syndrome
D. Benign lymphoepithelial lesion
E. Acute epidemic parotitis
Which of the following best indicates that a removable partial denture needs to be relined?
A. There is a loss of retention.
B. There is soreness on the crest of the ridges.
C. The indirect retainers are not seated as the extension bases are depressed.
D. The acrylic resin teeth are abraded and the occlusal vertical dimension is decreased.
Which of the following is the most common cause of intracapsular restraint of mandibular
movement?
A. Infection
B. Ankylosis
C. Disc interference disorders
A patient will receive a free gingival graft. The graft epithelium will undergo which of the
following alterations?
A. Dysplasia
B. Degeneration
C. Proliferation
D. Orthokeratinization
E. Formation of keratohyalin granules

Which of the following is most likely to cause a yellowish discoloration in a primary maxillary
incisor following trauma?
A. Blood pigments
B. Pulp hyperemia
C. Internal resorption
D. Pulp necrosis
E. Pulp chamber calcification
A patient is allergic to both amide and ester anesthetic derivatives. For this patient, the dentist
should find which of the following infiltrative local anesthetics to be safe and effective?
A. Bupivacaine
B. Phenylephrine
C. Nitrous oxide
D. Diphenhydramine
E. Ethylaminobenzoate
A patient presents with slight chipping of the enamel along the incisal edges of Teeth #8 and #9.
When choosing between restoring the incisal edges or reshaping by selective grinding, which of
the following important factors should the dentist consider?
A. Location of proximal contacts
B. Shape of incisal embrasures
C. Amount of translucent enamel present
Children and adolescents are the LEAST likely to develop which of the following?
A. Ameloblastic fibroma
B. Calcifying epithelial odontogenic tumor
C. Adenomatoid odontogenic tumor
D. Compound odontoma

A dentist is restoring a patient's mandibular arch with a removable partial denture (RPD). The
RPD will replace the second premolars and all molars on both sides. Which of the following is
the best method for recording centric relation?
A. Use a plaster record of the interdigitation of teeth
B. Manually articulate the casts and secure with sticky wax
C. Use the framework that has an occlusal rim attached
D. Use a wax registration that covers premolars of the mandibular arch and anterior teeth
E. Use an occlusal rim made on the master cast to which is added a soft wax for the
registration
The Occupational Safety and Health Administration (OSHA) is concerned with regulated waste
within the office. The Environmental Protection Agency (EPA) regulates the transportation of
waste from the office.
A. Both statements are TRUE.
B. Both statements are FALSE.
C. The first statement is TRUE, the second is FALSE.
D. The first statement is FALSE, the second is TRUE.
A 6-year-old patient has a dark brown discoloration of his maxillary central incisor. The
discoloration started following trauma accompanied with a facial sinus tract. What is the
treatment-of- choice?
A. Extraction
B. Pulpectomy
C. Pulpotomy
D. Direct pulp treatment
Squamous cell carcinoma appears in a variety of locations. In which of the following locations is
its prognosis the LEAST favorable?
A. Lower lip
B. Floor of the mouth

C. Hard palate
D. Buccal mucosa
Although a dentist wishes to use the long-cone paralleling technique (40 cm source-object
distance), the x-ray machine is equipped with only a short cone (20 cm PID). As a result, the
dentist moves the machine away from the patient's face so the source-object distance is 40 cm
and adjusts the exposure time appropriately. Compared to the degree of tissue covered by an
actual long cone, this "extended" short cone exposes what amount of tissue?
A. More
B. The same
C. Less
Fluoride therapy and occlusal sealants modify which of the following four factors the most?
A. Host
B. Time
C. Substrate
D. Microflora
Which of the following drugs produces sufficient central nervous system depression to cause a
state of sleep from which one may be aroused?
A. A sedative
B. A hypnotic
C. An opiate
D. An anesthetic
Which of the following usually causes the slow progress in molar uprighting in an adult patient?
A. Appliance friction
B. Excessive deep bite
C. Occlusal interferences
D. Poor patient cooperation

E. Lack of anchorage control


Squamous cell carcinoma of the tongue MOST commonly metastasizes to which of the
following?
A. Lungs
B. Liver
C. Bones
D. Cervical nodes
To reduce a patient's salivary flow, a dentist has prescribed atropine. As a result of this
medication, the patient might experience which of the following side effects?
A. Sedation
B. Diarrhea
C. Bradycardia
D. Blurred vision
E. Stomach cramping
Which of the following is the OPTIMAL reduction for the lingual cusp on Tooth #3 to receive an
MODL onlay?
A. 1.0 mm
B. 1.5 mm
C. 2.0 mm
D. 2.5 mm
Of the following local anesthetics, which has intrinsic vasoconstrictive actions?
A. Cocaine
B. Procaine
C. Xylocaine
D. Bupivacaine

During an interview, a dentist asks a patient, "You are not afraid of dentistry, are you?" This is an
example of which type of question?
A. Leading
B. Closed
C. Open
D. Funneling
To be effective in translating the roots of teeth, an orthodontic appliance must be
A. very gentle in action.
B. augmented with extraoral force.
C. capable of exerting a torque or moment.
D. capable of exerting positive, intermittent forces.
Which of the following conditions can cause delayed eruption of permanent teeth?
A. Cherubism
B. Hyperthyroidism
C. Hyperparathyroidism
D. Paget's disease of bone
Class I carious lesions are the LEAST likely to occur on which of the following?
A. Lingual surfaces of mandibular incisors
B. Lingual surfaces of maxillary incisors
C. Lingual surfaces of maxillary molars
D. Facial surfaces of mandibular molars
Inhalation of 100% oxygen is contraindicated for a person who has
A. a flail chest.
B. pneumothorax.

C. acute bronchitis.
D. acute viral pneumonia.
E. chronic obstructive lung disease.
Which of the following antibiotics is found at much higher concentrations in crevicular fluid than
in serum?
A. Clindamycin
B. Penicillin
C. Metronidazole
D. Tetracycline
Which of the following exhibits neoplasia of both the epithelial and connective tissue
components?
A. Ameloblastoma
B. Cementoblastoma
C. Ameloblastic fibroma
D. Odontogenic adenomatoid tumor
Non-working interferences usually occur on the inner aspects of the
A. facial cusps of maxillary molars.
B. facial cusps of mandibular molars.
C. lingual cusps of mandibular molars.
D. facial cusps of maxillary premolars.
A vital permanent tooth has an irreversible pulpitis and an open apex. Which of the following
represents the treatment-of-choice for this tooth?
A. Formocresol pulpotomy
B. Indirect pulp therapy
C. Calcium hydroxide pulpotomy

D. Conventional root canal therapy


E. Apexification
Each of the following is associated with Sjogren's syndrome EXCEPT one. Which one is the
EXCEPTION?
A. Arthritis
B. Xerostomia
C. Cervical caries
D. Warthin's tumor
E. Keratoconjunctivitis sicca
A permanent tooth with a mature apex has become intruded deeply into the bone. As a result of
this trauma, which of the following pulpal changes is the most likely to occur?
A. Necrosis
B. Internal resorption
C. Calcific metamorphosis
D. Transient inflammation
Trigeminal neuralgia (tic douloureux) is characterized by
A. paralysis of one side of the face.
B. uncontrollable twitching of one eye.
C. sharp, excruciating pain of short duration.
D. prolonged episodes of pain in one side of the face.
E. dull pain when pressure is applied over the affected area.
An individual has just received the first in a series of three hepatitis B vaccines. For the
seroconversion of this series to be the most effective, the person should receive the second and
third vaccinations how many months later?
A. Second vaccination one month later, third vaccination three months later

B. Second vaccination one month later, third vaccination six months later
C. Second vaccination two months later, third vaccination three months later
D. Second vaccination three months later, third vaccination six months later
Each of the following statements describes intracanal instruments and their action EXCEPT one.
Which one is this EXCEPTION?
A. A K-type file can be used in a filing and reaming action, while a reamer can be used in
only a reaming action.
B. Clockwise-turning of a root canal instrument will force debris in an apical direction,
while counterclockwise-turning will cause debris to move in a coronal direction out of
the canal.
C. A reaming action will enlarge a root canal and produce a cross-sectional preparation that
is approximately round in shape.
D. An intracanal instrument is more susceptible to breakage if locked in dentin and then
rotated.
The usual metabolic pathway of ingested fluoride involves urinary excretion primarily, with the
remaining portion found largely in
A. teeth.
B. muscle tissues.
C. skeletal tissues.
D. epidermal tissues.
Pain that has no organic basis and that is fixed upon some portion of the anatomy is usually
referred to as
A. false.
B. psychogenic.
C. referred.
D. phantom.
In MOST instances, intraoral verruca vulgaris develops as the result of

A. trauma.
B. pipe smoking.
C. autoinoculation.
D. chronic alcoholism.
What happens to the permanent molar occlusion in the presence of a flush (straight) terminal
plane and mandibular primate spaces?
A. Erupts end-to-end; early mesial shift into Class I occlusion
B. Erupts end-to-end; late mesial shift into Class I occlusion
C. Erupts with Class II tendency
D. Erupts with Class III tendency
A patient presents for try-in evaluation of balanced occlusion of complete maxillary and
mandibular dentures. A dentist notes that protrusive excursion results in separation of posterior
teeth. This dentist can best correct this problem by
A. changing the condylar inclination.
B. increasing the incisal guidance.
C. increasing the compensating curve.
D. using a flat plane cusp for the posterior teeth.
Which of the following mucocutaneous disorders is characterized by (1) degeneration of the
basal cell layer, (2) "saw-tooth" rete pegs, and (3) a dense mononuclear inflammatory-cell
infiltrate in the subepithelial connective tissue?
A. Lichen planus
B. Erythema multiforme
C. Keratosis follicularis
D. Dermatitis herpetiformis
E. Benign mucous membrane pemphigoid

An individual's mandible is growing forward less than the maxilla. Each of the following will be
an appropriate treatment strategy EXCEPT one. Which one is this EXCEPTION?
A. Stimulating the mandibular growth
B. Inhibiting the maxillary growth
C. Redirecting the maxillary growth
D. Retracting the mandibular teeth
E. Retracting the maxillary teeth
Nausea and vomiting that are associated with administration of opioid analgesics is the result of
stimulation of the
A. limbic system.
B. emetic center.
C. chemoreceptor trigger zone (CTZ).
D. opioid receptors in the G.I. tract.
Orange stain is used to
A. change the hue of porcelain color.
B. increase the interproximal translucency.
C. increase the value of porcelain color.
D. decrease the chroma of porcelain color.
Vesicular lesions precede the formation of ulcers in each of the following EXCEPT one. Which
one is this EXCEPTION?
A. Herpangina
B. Herpes zoster
C. Herpetic stomatitis
D. Aphthous stomatitis
E. Hand-foot-and-mouth disease

A properly trimmed wooden wedge will do each of the following EXCEPT one. Which one is
this EXCEPTION?
A. Protect the gingival tissue
B. Provide space for the matrix band
C. Prevent overcontouring of the contact area
D. Reduce moisture leakage into the cavity preparation
When is the appropriate time to correct a crossbite of a permanent maxillary central incisor?
A. When the tooth is erupting
B. After the tooth is fully erupted
C. After all anterior teeth are erupted
D. After the opposite central incisor has erupted
Parallelism of abutment preparations is determined by the
A. volume of chamfer.
B. degree of convergence.
C. angulation of finish lines.
D. long axis of the preparations.
E. long axis of natural teeth.
A patient experiences numbness of the left upper lip, cheek, and the left side of the nose
following a fracture of his midface. This symptom follows a fracture through the
A. nasal bone.
B. zygomatic arch.
C. maxillary sinus.
D. infraorbital rim.
When is the appropriate time to take the first bite-wing radiographs of a clinically caries-free
child?

A. When spaces between the posterior teeth have closed


B. When the entire primary dentition is completely erupted
C. When the permanent first molars have erupted
D. At the first dental examination
Which of the following describes primary occlusal trauma?
A. It is the first incidence of trauma that a tooth experiences.
B. It is a trauma that produces irreversible damage to the periodontium.
C. Mobility is caused by excessive forces on a normal periodontium.
D. Mobility is caused by excessive forces on a reduced periodontium.
The maximal or "ceiling" effect of a drug is also correctly referred to as the drug's
A. agonism.
B. potency.
C. efficacy.
D. specificity.
Which condition is characterized by lesions in the central nervous system that manifests itself as
various types of neuromuscular dysfunction?
A. Cerebral palsy
B. Down syndrome
C. A learning disability
D. Hyperkinesis (attention deficit disorder)
In a restorative problem involving all teeth in the mouth, the protrusive condylar path inclination
has a primary influence on the
A. incisal guidance.
B. anterior teeth only.

C. mesial inclines of the mandibular cusps.


D. mesial inclines of the maxillary cusps.
In mg/kg body weight, the lethal dose of fluoride falls in the range of
A. 0.5-1.0.
B. 20-50.
C. 100-200.
D. 300-500.
Which of the following teeth are the least likely to have more than one canal?
A. Maxillary lateral incisors
B. Mandibular lateral incisors
C. Mandibular first premolars
D. Maxillary second premolars
E. Maxillary second molars
Child abuse/neglect most commonly involves children in which age group?
A. Birth to age three years
B. Four to six years
C. Seven to nine years
D. All age groups are equally affected.
A dentist will remove a mandibular lingual torus from a patient's premolar region. This dentist
should use an envelope flap design that
A. has no vertical components.
B. has an anterior vertical component only.
C. has anterior and posterior vertical components.
D. is incised at the junction of attached gingival tissue and free mucosa.

Each of the following will assist a dentist in diagnosing or assessing Sjogren's syndrome
EXCEPT one. Which one is this EXCEPTION?
A. Obtaining sialograms
B. Performing exfoliative cytology
C. Obtaining blood samples for serology
D. Determining salivary flow rate
E. Obtaining labial gland biopsy
The dentist tried-in the metal framework for a porcelain fused-to-metal crown and the margins
were closed. When the completed crown was returned from the lab, the margins are all open. The
most likely reason for this is the
A. die was overtrimmed.
B. lab cut off the margins.
C. casting distorted during the porcelain application.
D. porcelain proximal contact areas are over-contoured
If the line of force were applied through a tooth's center of resistance, the tooth would
A. tip.
B. rotate.
C. intrude.
D. translate.
E. extrude.
Intensifying screens are used with extraoral radiographic films to
A. increase kVp.
B. increase exposure time.
C. improve image quality.
D. decrease radiation to the patient.

An edentulous patient has mobile, hyperplastic tissue in her maxillary anterior region. In making
impressions for this patient, the dentist should
A. use the closed-mouth technique.
B. use a high-fusing impression compound.
C. register the tissue in its passive position.
D. involve maximum pressure.
A patient's permanent tooth crown fractures, creating a small (1 mm) pulp exposure for about 30
minutes. Which of the following pulp therapies is the most appropriate for this patient?
A. Pulpectomy and apexification, if necessary
B. Direct pulp capping with calcium hydroxide
C. Pulpotomy with formocresol
D. Pulpotomy with calcium hydroxide
A 50-year-old man complains of a burning, aching pain on the side of his face. He experiences
paresthesia and itching in the same area. An examination reveals small white scars in the preauricular region. The MOST probable diagnosis is
A. Bell's palsy.
B. psychoneurosis.
C. post-herpetic neuralgia.
D. auriculotemporal syndrome.
Prolonged, unstimulated night pain suggests which of the following conditions of the pulp?
A. Pulp necrosis
B. Mild hyperemia
C. Reversible pulpitis
D. No specific condition

Some teeth appear to be clinically normal, but exhibit (1 ) globular dentin, (2) very early pulpal
obliteration, (3) defective root formation, (4) periapical granulomas and cysts, and (5) premature
exfoliation. The condition is known as which of the following?
A. Shell teeth
B. Dentin dysplasia
C. Regional odontodysplasia
D. Amelogenesis imperfecta
E. Dentinogenesis imperfecta
When a nonrigid connector is used in a fixed partial denture, the path of insertion of the key into
the keyway should be parallel to the path(s) of insertion of
A. the retainer carrying the keyway.
B. the retainer not involved with the keyway.
C. both retainers.
Green and orange stains on maxillary incisors can usually be attributed to
A. drugs.
B. diet.
C. poor oral hygiene.
D. fluoride consumption.
The best measure of the potential clinical performance of a casting alloy is its
A. castability.
B. burnishability.
C. ADA certification.
D. tarnish susceptibility.
E. mechanical properties.
Which of the following cysts is the LEAST likely to be visible radiographically?

A. Nasolabial
B. Nasopalatine
C. Lateral periodontal
D. Traumatic bone
Which drug group is the least likely to cause xerostomia?
A. Opioids
B. Antidepressants
C. Antihistaminics
D. Benzodiazepines
E. Anticholinergics
Which of the following is measured by the modulus of elasticity?
A. Stiffness or rigidity
B. Ultimate strength
C. Yield strength
D. Ductility or malleability
Which of the following represent(s) the serious effects of a repeated exposure to low doses of X
radiation?
A. Purpura
B. Carcinogenesis
C. Diarrhea and dehydration
D. Alteration of the oral microflora
Currently, the best oral sedative drugs for dentistry fall into the class of
A. narcotics.
B. barbiturates.

C. phenothiazines.
D. benzodiazepines.
Which of the following best describes the drug-receptor activity of naloxone?
A. High affinity, high intrinsic activity
B. Low affinity, high intrinsic activity
C. High affinity, no intrinsic activity
D. No affinity, low intrinsic activity
The amalgam preparation outline for the primary mandibular second molar closely resembles
that for which permanent tooth?
A. Maxillary second premolar
B. Mandibular first premolar
C. Mandibular second premolar
D. Mandibular first molar
The rate at which new disease occurs is classified as which of the following?
A. Incidence
B. Prevalence
C. Extent
D. Attributable risk
Which of the following conditions indicates that a periodontal, rather than an endodontic
problem, exists?
A. Acute pain to percussion with no swelling
B. Pain to lateral percussion with a wide sulcular pocket
C. A deep narrow sulcular pocket to the apex with exudate
D. Pain to palpation of the buccal mucosa near the tooth ape

An 82-year-old woman presents with a large four-surface pin-retained amalgam restoration on


Tooth #3. The dentist notes minor recurrent caries along the faciocervical amalgam margin.
Which of the following is the treatment-of-choice?
A. Replace the restoration
B. Repair the defect
C. Prepare the tooth for a crown
D. Observe at recall
The proximal portion of a Class II cavity preparation in a primary molar extends rather deep
gingivally. A satisfactory gingival seat may NOT be obtained because of which of the following?
A. The primary teeth have a marked cervical constriction.
B. The proximal contact of primary molars is broad and flat.
C. The facial and lingual surfaces of primary molars converge occlusally.
D. The enamel rods in the gingival third of the primary first molars extend occlusally.
During which stage of tooth development is the cariostatic effect of fluoride manifested?
A. Apposition
B. Calcification
C. Proliferation
D. Histodifferentiation
The light source affects the perception of color, BECAUSE the light source must contain the
wavelength of the color to be matched in order to see that color.
A. Both the statement and the reason are correct and related.
B. Both the statement and the reason are correct but NOT related.
C. The statement is correct, but the reason is NOT.
D. The statement is NOT correct, but the reason is accurate.
E. NEITHER the statement NOR the reason is correct.

Orthodontic correction of which of the following is the most easily retained?


A. Diastema
B. Rotation
C. Expansion
D. Anterior crossbite
E. Generalized spacing
Which of the following types of chemical bonding is the least likely to be involved in a drugreceptor interaction?
A. Covalent bonding
B. Hydrogen bonding
C. Dipole-dipole bonding
D. Electrostatic bonding
E. van der Waal's forces
Which of the following best explains why the renal excretion of salicylic acid is increased in an
alkaline urine?
A. The urine volume is decreased.
B. The percentage of salicylic acid that is ionized in urine is increased.
C. The percentage of salicylic acid that is ionized in urine is decreased.
D. Decreasing urine pH usually increases the renal excretion of weak acids.
X-rays are produced when
A. protons strike the anode.
B. electrons strike the anode.
C. the anode is heated above 3,000 degrees Centigrade.
D. the filament becomes positively charged.

A member of the dental staff routinely fails to clean up his work area. Which of the following
best exemplifies an assertive message that could be made by a co-worker?
A. If you refuse to do your part, I'm just going to have to report the situation to the boss.
B. When you don't help with the clean-up, it makes me angry because I have to do both your
work and my own.
C. I'm tired of cleaning up your mess!
D. I hope you'll remember to help me clean up today.
E. Is something bothering you? You sometimes forget to clean up.
The crown of an endodontically-treated maxillary lateral incisor is fractured near the gingival
margin. The coronal end of the silver cone used in filling that canal is visible at that level. The
findings reveal that the existing root canal filling meets all criteria to be judged successful.
Which of the following is the best way to obtain the needed post space?
A. To prepare it alongside the silver cone using burs and Peeso reamers.
B. To carefully grind away the coronal part of the silver cone using round burs or endcutting burs.
C. To remove the silver cone and re-treat the canal using a gutta-percha technique, and then
to create the needed post space.
D. To remove the silver cone, notch it 4 mm from the apical end, coat it with freshly mixed
sealer, replace it in the canal, and "twist-off" the coronal segment.
After receiving one cartridge of a local anesthetic, a healthy adult patient became unconscious in
the dental chair. The occurrence of a brief convulsion is
A. pathognomonic of grand mal epilepsy.
B. consistent with a diagnosis of syncope.
C. usually caused by the epinephrine in the local anesthetic.
D. pathognomonic of intravascular injection of a local anesthetic
The axial wall of an occlusolingual amalgam preparation on a maxillary molar should be in
dentin and
A. parallel to the dentinoenamel junction.

B. parallel to the long axis of the tooth.


C. at an acute angle with the pulpal floor.
During the mixed dentition stage, which of the following appliances should be used as a space
maintainer for missing primary molars in the mandibular arch?
A. Distal shoe
B. Nance holding arch
C. Passive lingual arch
D. Removable acrylic functional
Accepted methods for closing a diastema between maxillary central incisors include using either
a removable appliance with finger springs or
A. a rubber elastic around the two teeth.
B. a steel ligature around the two teeth.
C. bonded brackets with intertooth traction.
Each of the following developmental cysts appears on radiographs EXCEPT one. Which one is
this EXCEPTION?
A. Nasolabial
B. Median palatal
C. Globulomaxillary
D. Nasopalatine
When trying-in a porcelain fused-to-metal crown, the dentist observes that the gingival-margin
finish-line integrity is excellent, but that the occlusal surface is 1 mm too high. Which of the
following is the most probable cause?
A. Incorrectly related casts
B. Proximal contacts being too tight
C. A distortion of the metal during firing
D. An expansion of the porcelain during firing

The Health Belief Model attempts to explain patient recommendations by


A. examining patients' perceptions of disease-seriousness and of the treatment effectiveness.
B. focusing on the doctor-patient relationship.
C. emphasizing the barriers that are external to behavior change, for example, payment
mechanisms.
D. focusing on patients' skills at carrying out recommendations.
Which of the following represents the MOST effective way of encouraging a child to floss
regularly?
A. Presenting a message that concerns the potential loss of teeth, should the patient not floss
B. Arranging for the child to earn privileges for flossing
C. Repeating periodically the message on the value of flossing
D. Providing written material that explains the mechanism of flossing
In an ideal Class V cavity preparation for amalgam in a mandibular premolar, retention form is
gained
A. into the mesial and distal walls.
B. in the mesial and occlusal line angles.
C. in the axio-occlusal and axiogingival line angles at the expense of the axial wall.
D. in the axio-occlusal and axiogingival line angles at the expense of the occlusal and
gingival walls.
Which of the following systems is thought to malfunction in the hereditary form of angioneurotic
edema?
A. C-1 esterase
B. C-1q inhibitor
C. CH50 consumption
D. Serine phosphatase
E. Complement synthetase

Following a bilateral mandibular fracture in the canine region, the anterior fragment of the
mandible is displaced posteriorly by the action of the geniohyoid and by which other muscles?
A. Thyrohyoid and mylohyoid only
B. Genioglossus and mylohyoid
C. Thyrohyoid, mylohyoid, and anterior bellies of the digastric
D. Genioglossus and anterior bellies of the digastric
The number of persons who die each year from oral cancer in the United States approximates
A. 2000
B. 4000
C. 8000
D. 16000
Each of the following parameters has an effect on the distribution of a drug EXCEPT one. Which
one is this EXCEPTION?
A. The intrinsic activity of the drug
B. The binding of a drug to plasma proteins
C. The blood flow to various organs and tissues
D. The relative solubility of the drug in the tissues
Which of the following is MOST appropriate for testing differences between the means of two
groups?
A. Chi-square test
B. Multiple regression analysis
C. Correlation coefficient analysis
D. Student's t-test
When a removable partial denture is terminally seated, the retentive clasp tips should
A. be invisible.

B. exert no force.
C. apply retentive force into the body of the teeth.
D. resist torque through the long axis of the teeth.
The decrease in ventilation caused by morphine and by some of the related opioids depends
chiefly upon a decrease in
A. blood PC02 concentration.
B. medullary endorphin activity.
C. sensitivity of the medullary respiratory center to carbon dioxide.
D. sensitivity of aortic and carotid chemoreceptors to blood oxygen concentration.
The depth of the clinical gingival sulcus is the distance from the gingival margin to the
A. cementoenamel junction.
B. most apical penetration of the periodontal probe.
C. most apical extension of the junctional epithelium.
D. bottom of the sulcus in well-preserved histologic block sections.
Additional space for successive eruption of permanent maxillary molars is provided by
A. interstitial bone growth.
B. appositional growth at the maxillary tuberosity.
C. continuous expansion of the dental arch due to sutural growth.
D. an increase in palatal vault height due to alveolar growth.
Which of the following responses made by the dentist is the MOST likely to increase a patient's
adherence to oral hygiene prescriptions?
A. You don't want lots of cavities, do you?
B. If you take my advice, you can save yourself pain and money.
C. You really should spend the time it takes to brush and floss.

D. What do you see as some of the problems with this approach?


E. I can assure you that this approach is quick and easy.
Amalgam scrap should be stored in a tightly-sealed container and covered with which of the
following?
A. Water
B. Glycerin
C. Sulfide solution
D. Sodium hypochlorite solution
The soft tissue-tooth interface that forms most frequently after flap surgery in an area previously
denuded by inflammatory disease is a
A. collagen adhesion.
B. reattachment by scar.
C. long junctional epithelium.
D. connective tissue attachment
Which of the following is inherited as an autosomal dominant trait?
A. Lichen planus
B. Bullous pemphigoid
C. Pemphigus vulgaris
D. White sponge nevus
E. Epidermolysis bullosa acquisita
Which of the following is the primary substance or material removed during root planing?
A. Stain
B. Dentin
C. Plaque

D. Calculus
E. Cementum
Three days after full mouth extraction, an elderly patient complains of black-and-blue marks on
her neck. The most probable diagnosis is
A. thrombocytopenia.
B. capillary fragility.
C. hematoma formation.
D. postoperative ecchymosis.
Most epidemiologic studies indicate that gingivitis in children is relatively common. A strong
positive association between specific nutritional deficiencies and the presence of periodontal
disease in children and adults has been demonstrated.
A. Both statements are TRUE.
B. Both statements are FALSE.
C. The first statement is TRUE, the second is FALSE.
D. The first statement is FALSE, the second is TRUE.
Chroma is that aspect of color that signifies the
A. degree of grayness.
B. degree of translucency.
C. degree of saturation of the hue.
D. combined effect of the hue and value.
Which of the following characterize(s) the histopathologic changes in chronic gingivitis?
A. The loss of rete pegs and the dissolution of the basement membrane
B. An inflammatory infiltrate in which macrophages predominate
C. An inflammatory infiltrate in which neutrophilic leukocytes and mast cells predominate
D. The destruction of the principal fibers of the periodontal ligament

E. The disruption of the gingival fibers and an inflammatory infiltrate of plasma cells,
lymphocytes, and neutrophilic leukocytes
Which of the following most strongly suggest cause-and-effect relationships?
A. Correlational studies
B. Controlled clinical trials
C. Case reports
D. Epidemiologic surveys
Spontaneous osteogenic sarcoma is associated with which of the following disease processes of
bone?
A. Paget's disease
B. Histiocytosis-X
C. Hyperparathyroidism
D. Letterer-Siwe disease
E. Osteogenesis imperfecta
The drug-of-choice for the treatment of adrenergically-induced arrhythmias is
A. quinidine.
B. lidocaine.
C. phenytoin.
D. propranolol.
Which of the following drugs, when administered intravenously, is LEAST likely to produce
respiratory depression?
A. Fentanyl
B. Diazepam
C. Thiopental
D. Meperidine

E. Pentobarbital
Which of the following safety techniques provides the GREATEST DECREASE in overall
radiation-risk to patients?
A. Changing from Group D to Group E film
B. Switching from round to rectangular collimation
C. Using an automatic rather than manual processing switch
D. Adding a cervical collar to a leaded apron
Premature exfoliation of a primary mandibular canine is most often the sequela of which of the
following?
A. Caries
B. Trauma
C. Serial tooth extraction
D. Arch length inadequacy
In lateral movements, the non-working condyle moves in what direction?
A. Laterally, posteriorly, and superiorly
B. Superiorly, posteriorly, and inferiorly
C. Inferiorly, anteriorly, and medially
D. Inferiorly, anteriorly, and laterally
A dentist who is planning to restore a tooth with gold alloy desires a certain amount of expansion
in the casting investment mold. Which of the following best explains why this expansion is
desirable?
A. It allows for the shrinkage of the molten metal and the wax pattern.
B. It compensates for the shrinkage of the molten metal as it cools in the mold.
C. It makes the metal casting larger than the wax pattern so that the casting will fit the tooth
tightly.

The dentist is using polyether material to make impressions. The physical property of this
material that should concern this dentist is the
A. coefficient of thermal conductivity.
B. hydrophilic nature.
C. extended setting time.
D. high modulus of elasticity.
Streptococcus mutans is considered to be a principal etiologic agent of caries because it produces
organic acids and it
A. forms a gelatinous matrix.
B. metabolizes substrate from saliva.
C. derives energy from enamel constituents.
D. lives symbiotically with Lactobacillus acidophilus.
Which of the following best explains how high-copper amalgam restorations differ from
conventional amalgam restorations? The high-copper restorations
A. have little or no tin-mercury phase.
B. require less mercury, so there is more matrix formed.
C. corrode at an accelerated rate due to increased copper content.
D. are unaffected by moisture contamination in the presence of zinc
What do Gardner syndrome and cleidocranial dysplasia have in common?
A. Intestinal polyps
B. Intraoral pigmentation
C. Impacted supernumerary teeth
D. Osteomas of the skull and jaws
A practitioner who is restoring a tooth with composites wishes to be sure that the matrix has
adhered to the filler. Which of the following agents adheres the resin matrix to the filler?

A. Wetting
B. Coupling
C. Catalyzing
D. Activating
An operator has chosen to use a shielded open-ended cone. Which of the following will
contribute the most to patient gonadal dose?
A. Leakage from the x-ray machine head
B. Scatter from the operatory walls
C. Scatter from the cone
D. Scatter from the patient's face
Which of the following provides the best guarantee for sterilization in a heat sterilizer?
A. Using a chemical indicator strip or pouch
B. Recording the temperature and/or pressure readings from the sterilizer gauges
C. Using bacterial spore tests
D. Determining the ability of the sterilizer to kill the hepatitis B virus
E. Demonstrating inactivation of the tuberculosis bacterium
The integrity of the floor of the antrum is at greatest risk with surgery involving the removal of
(a)
A. torus palatinus.
B. nasopalatine cyst.
C. maxillary third molar.
D. single remaining maxillary molar.
E. supernumerary teeth from the maxillary canine region.
Which of the following is the principal nonverbal cue that two or more people can use to regulate
verbal communication?

A. Posture
B. Movement
C. Eye contact
D. Body position
When the mandible is in its physiologic rest position, the distance between the occluding
surfaces of the maxillary and mandibular teeth or occlusion rims is
A. vertical dimension.
B. interocclusal distance.
C. vertical dimension of rest.
D. centric occlusion (maximum intercuspation).
The practitioner would like to learn whether or not a patient understands the treatment plan.
Which of the following questions is the MOST likely to elicit this information?
A. Do you understand what I mean?
B. Do you have any questions about this approach?
C. What benefits or drawbacks do you see in this approach?
D. Is this approach OK with you?
The distolingual extension of a mandibular impression for a complete denture is limited by the
action of which of the following muscles?
A. Stylohyoid
B. Lateral pterygoid
C. Medial pterygoid
D. Superior constrictor
A defect in neuromuscular transmission causes which of the following?
A. Bell's palsy
B. Myasthenia gravis

C. Muscular dystrophy
D. Multiple sclerosis
E. Trigeminal neuralgia
The day after receiving an inferior alveolar nerve block, a patient experiences limited ability to
open his mouth. Which of the following structures was most probably injured?
A. The medial pterygoid muscle
B. The stylomandibular ligament
C. The deep fibers of the masseter muscle
D. The posterior belly of the digastric muscle
E. The inferior head of the lateral pterygoid muscle
Lesions of recurrent herpetic stomatitis are usually found mainly on
A. loosely attached areas of the oral mucosa, such as the buccal mucosa and the floor of the
mouth.
B. tightly attached areas of the oral mucosa, such as the hard palate and the attached gingiva.
C. loosely attached areas of the oral mucosa, such as the soft palate and
D. the alveolar mucosa.
E. tissue anywhere in the oral mucosa, including both loosely- and tightly-attached areas.
Which of the following instruments should be used to plane the facio-proximal cavosurface
margin of a standard Class II preparation on a mandibular molar?
A. Straight chisel
B. Binangle chisel
C. Enamel hatchet
D. Bibeveled hatchet
Which of the following most accurately describes the effect of individual plaque control
instructions that are conducted in the classroom?

A. A short-term decrease in gingivitis


B. A long-term decrease in gingivitis
C. A short-term decrease in caries
D. A long-term decrease in caries
An endodontic instrument separated in the apical third of a root canal. The fragment is 3 mm
long and is tightly lodged. No radiographic changes at the apex are evident. The practitioner
should
A. extract the tooth.
B. resect the apical section of the root containing the broken instrument.
C. perform an apicoectomy and place a reverse filling.
D. complete the root canal filling to the level of the instrument and observe.
A new patient had root canal therapy performed seven months ago in another country. No
historical radiographs are available. The root canal filling appears to be satisfactory, the tooth is
asymptomatic, and there is no associated sinus tract. However, a small periapical radiolucency is
evident. Which of the following is indicated?
A. Incision and drainage
B. Nonsurgical retreatment
C. Re-evaluation in six months
D. Apicoectomy and apical amalgam
E. Prescription of an appropriate antibiotic
Which of the following do polycarboxylate and glass ionomer have in common?
A. Zinc oxide
B. Polysiloxane
C. Phosphoric acid
D. Polyacrylic acid
E. Ion-leachable glass

A practitioner has burnished a margin. As a result of this procedure, the dental casting gold has
become stronger, but less ductile. Which of the following explains why?
A. The gold has work-hardened.
B. The grains have elongated.
C. The stiffness has increased.
D. The surface flaws have been removed.
The light from visible-fight polymerization units can cause which of the following?
A. Iritis
B. Cataracts
C. Retinal damage
D. Corneal ulcerations
Which of the following responses made to a child is MOST likely to reinforce positively the
child's appropriate behavior in the dental setting?
A. You're doing a great job for me today.
B. I like it when you sit quietly in the chair.
C. This visit is going really well, don't you think?
D. Coming to the dentist isn't so bad after all, is it?
An endodontically-treated permanent mandibular first molar has incipient lesions on its mesial
and distal surfaces. During previous treatment, a minimal amount of tooth structure was
removed. The appropriate treatment for this tooth is a (an)
A. MOD amalgam.
B. MOD cast gold inlay.
C. MOD cast gold onlay.
D. 3/4 crown.
E. full crown.

A patient has a skeletal deformity with a Class III malocclusion. This deformity is the result of a
maxillary deficiency. The treatment-of -choice is
A. orthodontics.
B. surgical repositioning of the maxilla.
C. anterior maxillary osteotomy.
D. posterior maxillary osteotomy.
E. surgical repositioning of the mandible.
The hamular notch is important in complete denture construction BECAUSE it aids the dentist in
positioning the maxillary posterior teeth.
F. Both the statement and the reason are correct and related.
G. Both the statement and the reason are correct but NOT related.
H. The statement is correct, but the reason is NOT.
I. The statement is NOT correct, but the reason is accurate.
J. NEITHER the statement NOR the reason is correct.
The most common type of active tooth movement in the primary dentition is correction of which
type of problem?
F. Vertical
G. Sagittal
H. Transverse
I. Space regaining in the maxillary arch
J. Space regaining in the mandibular arch
A patient has a chronically tender, knife-edge mandibular residual ridge. In fabricating a
complete denture for this patient, a dentist should consider which of the following?
E. Maximal extension of the denture--to distribute forces of occlusion over a greater area
F. Minimal extension of the denture--to limit tenderness to a smaller area

G. Decreased occlusal vertical dimension--to decrease biting forces


H. A broad occlusal table--to provide a firmer contact in eccentric jaw relations
A patient presents with spontaneous necrotizing ulcers of the oral cavity. The WBC is 1,986,
while the differential CBC reveals lymphocytes--68%, monocytes--28%, polymorphonuclear
leukocytes--2%, eosinophils--l%, and basophils--l%. Which of the following diagnoses is the
MOST appropriate for this patient?
E. Leukemia
F. Infectious mononucleosis
G. Agranulocytosis
H. Recurrent aphthous ulcers
Individuals who are cognitively disabled (e.g. those who have Down syndrome) sometimes have
difficulty accepting dental care. In attempting to improve their cooperation, the dentist should do
which of the following as an initial therapy?
E. Administer sedative agents
F. Use graduated exposures to the dental setting
G. Use physical restraints
H. Refer the patient to a special care clinic
Oral hygiene improvement in the presence of systemic or nutritional deficiencies affects the
incidence of chronic inflammatory periodontal disease. It also affects the severity of chronic
inflammatory periodontal disease.
E. Both statements are TRUE.
F. Both statements are FALSE.
G. The first statement is TRUE, the second is FALSE.
H. The first statement is FALSE, the second is TRUE.
A patient who has which of the following conditions is most likely to have postoperative
bleeding after multiple extractions?
F. Angina

G. Diabetes
H. Cirrhosis
I. Rheumatic fever
J. Chronic bronchitis
Two adjacent cavities involving proximal contact can be prepared and restored with composite
resin at one appointment for each of the following reasons EXCEPT one. Which one is this
EXCEPTION?
D. Restoration of contact is enhanced.
E. Access to adjacent cavities is simplified.
F. Color matching is easier.
At birth, the palate is relatively flat; in adults, it is vault-shaped. By which of the following does
this change occur?
E. Bone resorption in the palatal vault
F. Growth of the maxillary sinuses
G. Deposition of the alveolar crestal bone
H. Bone deposition on the posterior wall of the maxillary tuberosity
A 50-year-old male presents with deep fibrotic pockets and angular bone loss. The dentist
reflects a flap, debrides the two and three-walled defects, and performs scaling and root planing.
Which procedure should the dentist do next?
E. Gingivectomy
F. Osseous grafts
G. Modified Widman flap
H. Apically repositioned flap
A patient who is on dicumarol therapy requires a tooth extraction. Which laboratory test is the
most valuable in evaluating this patient's surgical risk?
F. Clotting time

G. Bleeding time
H. Prothrombin time
I. Sedimentation rate
J. Complete blood cell count
In radiobiology, the "latent period" represents the period of time between
E. cell rest and cell mitosis.
F. the first and last dose in radiation therapy.
G. film exposure and image development.
H. radiation exposure and onset of symptoms.
A broad understanding of the development of human behavior requires knowledge of the basic
concepts of
E. maturation and learning.
F. dependence and independence.
G. gender and race.
H. generalization and facilitation.
What is the pH threshold level at which enamel demineralization occurs?
D. 6.5
E. 5.5
F. 4.5
Which of the following clinical lesions MOST often reveals histologic evidence of severe
dysplasia or carcinoma in situ?
F. Leukoedema
G. Leukoplakia
H. Lichen planus

I. Erythroplakia
J. White sponge nevus
Which of the following planes of space are used to classify malocclusion?
E. Horizontal, vertical, diagonal
F. Antero-posterior, sagittal, vertical
G. Antero-posterior, transverse, vertical
H. Antero-posterior, sagittal, coronal
Before adhesion can take place between a liquid and a solid, it is essential that the liquid surface
E. provide some mechanical interlocking with the solid.
F. exhibit a large contact angle with the solid.
G. enter into some form of chemical reaction with the solid.
H. exhibit a small contact angle with the solid.
Which of the following root surfaces is the most likely to be strip-perforated during canal
instrumentation of the mesial root of a mandibular first molar?
E. Facial
F. Lingual
G. Mesial
H. Distal
To increase the anterior mandibular vestibular depth, the practitioner places the mucosal graft
directly on the
E. mucosa.
F. periosteum.
G. muscle.
H. bone.

Which of the following generally describes children with defiant behavior?


E. They are usually around 3 years of age or younger.
F. They are also referred to as stubborn or spoiled.
G. They usually exhibit tremors in voice when speaking.
H. They exhibit quiet sobbing with profuse tears.
A patient has a mandibular molar with pulp necrosis, pain to percussion, and with no
periradicular lesions. The emergency treatment. for this tooth is which of the following?
F. Opening the tooth, performing apical trephination, temporizing, and checking the
occlusion
G. Prescribing analgesics and antibiotics and rescheduling the patient
H. Performing pulpotomy, temporizing, and checking the occlusion
I. Debriding the canals, temporizing, and checking the occlusion
J. Opening the tooth for drainage and leaving the tooth open
Local anesthetics aid in reducing the flow of saliva during operative procedures by
E. blocking the cholinergic nerve endings.
F. blocking innervation to major salivary glands.
G. blocking efferent parasympathetic nerve pathways.
H. reducing sensitivity and anxiety during tooth preparation.
In a complete maxillary denture, accurate adaptation of the border of the maxillary facial flange
affects which of the following the most?
E. Speech
F. Support
G. Stability
H. Esthetics
Fixing solution functions to

E. accelerate the film processing.


F. remove the atomic silver.
G. remove undeveloped silver salts.
H. reduce silver ions to metallic silver.
Which of the following patients should preferably be hospitalized for the oral surgery procedure
indicated?
E. A patient with a prosthetic cardiac valve, who requires a single tooth removed, and an
alveoloplasty procedure.
F. A severely retarded patient, who requires full mouth gingivectomy for treatment of
phenytoin-induced hyperplasia.
G. A patient on anticoagulant therapy whose present prothrombin time is 1.25 times the
control time, who requires removal of a single tooth.
H. A juvenile diabetic whose disease is controlled by daily insulin injections, who requires
removal of a chronically infected tooth.
A dentist uses an unsaturated, chemical sterilizer for sterilizing dental instruments. To effectively
kill all microorganisms and spores, this dentist must maintain a temperature of 132 degrees
Celsius at 20 - 40 psi for at LEAST how many minutes?
E. 10
F. 20
G. 30
H. 40
There are four basic stages in the pathogenesis of the periodontal lesion. In progressing order,
they are
E. initial, established, early, and advanced.
F. early, incipient, established, and advanced.
G. initial, early, established, and advanced.
H. incipient, early, established, and advanced.

Which of the following combinations of agents would be necessary to block the cardiovascular
effects produced by the injection of a sympathomimetic drug?
F. Atropine and prazosin
G. Atropine and propranolol
H. Prazosin and propranolol
I. Phenoxybenzamine and curare
J. Amphetamine and propranolol
A cleft lip occurs following the failure of permanent union between which of the following?
F. The palatine processes
G. The maxillary processes
H. The palatine process with the frontonasal process
I. The maxillary process with the palatine process
J. The maxillary process with the frontonasal process
The highest incidence of fibrous gingival enlargement (hyperplasia) is related to
F. puberty.
G. diabetes.
H. leukemia.
I. pregnancy.
J. medication.
A patient has two or more of her teeth joined only by cementum. This phenomenon is known as
F. fusion.
G. hypercementosis.
H. gemination.
I. dilaceration.

J. concrescence.
While the dentist is preparing a large carious lesion in Tooth #30 for a restoration, a pulp
exposure occurs. The patient angrily shouts at the dentist, "You incompetent 'creep'--you're
responsible for this problem!"- Of the following possible responses the dentist could make,
which one is the most emphatic?
F. Calm down, I can still restore your tooth adequately.
G. Not when I'm preparing a tooth with caries like you had.
H. I can see that you're very upset. You thought the tooth could be restored and now this
problem has occurred.
I. If you took care of your mouth the way you should, I wouldn't have been close to the
pulp.
J. I'm sorry this happened, but we must get on with the procedure.
Which of the following is the most serious toxic effect that is associated with overdose of
acetaminophen?
F. Hemorrhage
G. Renal necrosis
H. Hepatic necrosis
I. Gastric ulceration
J. Respiratory alkalosis
A patient needs relining of both maxillary complete denture and mandibular distal-extension
partial denture. The recommended procedure is to
E. make impressions of both simultaneously.
F. start first on the maxillary complete denture.
G. start first with the least stable prosthesis.
H. relate first the mandibular partial denture frame.
In which of the following do adenomatoid odontogenic tumors MOST often occur?
E. Anterior maxilla

F. Posterior maxilla
G. Anterior mandible
H. Posterior mandible
Duration of treatment regimens affect patient compliance. The severity of disease perceived by
the patient affects patient compliance.
E. Both statements are TRUE.
F. Both statements are FALSE.
G. The first statement is TRUE, the second is FALSE.
H. The first statement is FALSE, the second is TRUE
The Centers for Disease Control (CDC) recommends that sterilizing units be monitored at which
of the following intervals?
E. Daily
F. Weekly
G. Bi-weekly
H. Monthly
An evaluation of which of the following represents the most important aspect in shade selection
(for the restoration to match an existing dentition)?
F. Hue
G. Value
H. Chroma
I. Size
J. Shape
A fracture through the angle of the mandible can result in an upward displacement of the
proximal fragment. Which of the following groups of muscles produces this movement?
E. Digastric and geniohyoid

F. Masseter, digastric, and lateral pterygoid


G. Masseter, temporal, and lateral pterygoid
H. Masseter, temporal, and medial pterygoid
During an emergency dental visit, a moderately retarded 4-year-old child resists physically and
cries excessively. In this instance, the dentist should use which of the following?
E. Physical restraints
F. General anesthesia
G. The hand-over-mouth exercise
H. Voice control
A practitioner is restoring the mesio-occlusal marginal ridge of a maxillary left second molar. If
the marginal ridge is higher than the adjacent tooth, then it can create a problem in which of the
following excursions?
E. Protrusive
F. Retrusive
G. Working
H. Non-working
Lymphangioma is MOST closely related to
E. hemangioma.
F. angiosarcoma.
G. cystic hygroma.
H. hemangiopericytoma.
According to the American Heart Association, which of the following prophylactic antibiotic
regimens is recommended for a 20-kg child who has congenital heart disease?
E. 1.0 gram amoxicillin orally one hour before the dental procedure and 500 mg 4 times a
day for 2 days postoperatively

F. 1.0 gram penicillin V orally one hour before the dental procedure and 500 mg orally 6
hours later
G. 1.0 gram amoxicillin one hour before the dental procedure and 500 mg orally 6 hours
later
H. 3.0 grams amoxicillin orally one hour before the dental procedure and 1.5 grams orally 6
hours later
When compared therapeutically to penicillin G, penicillin V has a
F. slower renal excretion.
G. more reliable oral absorption.
H. broader antibacterial spectrum.
I. greater resistance to penicillinase.
J. lower potential for allergic reaction.
Which of the following teeth are usually affected by nursing-bottle caries?
E. Maxillary incisors
F. Mandibular molars
G. Mandibular incisors
H. Maxillary molars
An adult healthy patient has a marked indurated swelling. He has a temperature of 100 degrees
Fahrenheit and has been in considerable pain for 24 hours. Percussion of the maxillary right
central incisor causes discomfort as does palpation at its apex. Pulp vitality tests are negative for
this tooth, while adjacent control teeth test within normal limits. A radiograph reveals that the
maxillary right central incisor has a deep unbased restoration and a widened apical periodontal
ligament space. Which of the following is the best emergency treatment for this patient?
E. Extracting the maxillary right central incisor
F. Administering an antibiotic and analgesic and initiating root canal therapy when
symptoms subside
G. Debriding the root canal of the maxillary right central incisor and prescribing antibiotics
and analgesics

H. Incising and draining the swelling and prescribing antibiotics and analgesics
A decrease in which of the following causes an increase in radiographic density?
E. mA
F. kVp
G. Object-film distance
H. Source-object distance
A patient who has a Class III malocclusion presents for treatment. Each of the following is a
helpful diagnostic aid to distinguish between maxillary retrusion and mandibular protrusion
EXCEPT one. Which one is this EXCEPTION?
E. Photography
F. Cephalometry
G. Study models
H. Clinical evaluation
A dentist will use a buccal coil spring to regain space for a mandibular second premolar. Which
of the following is an undesirable side effect that is most commonly associated with this
procedure?
F. Pain
G. Gingival irritation
H. Severe mobility of the tooth
I. Tendency for the first molar to intrude
J. Tendency for the first premolar to rotate
Tooth bleaching affects a color change in which of the following tooth parts?
E. Dentin only
F. All of the enamel
G. Both dentin and enamel

H. Only the surface of enamel


In primary teeth, which, of the following most often causes calcium hydroxide pulpotomies to
fail?
E. Pulp calcification
F. Periapical pathosis
G. Internal resorption
H. Formation of dentin bridges
Each of the following diseases has been associated with the Epstein-Barr virus EXCEPT one.
Which one is this EXCEPTION?
E. Burkitt's lymphoma
F. Oral hairy leukoplakia
G. Infectious mononucleosis
H. Adenoid cystic carcinoma
In a 12-year-old, an ANB angle of 2 degrees usually signifies that the
E. relationship of the maxilla to the mandible is favorable.
F. relationship of the mandible to the cranium is favorable.
G. cranial growth is poor.
H. incisors are upright.
Which of the following mechanical properties limits the amount of adjustment of a base metal
removable clasp arm?
E. Hardness
F. Stiffness
G. Elongation
H. Tensile strength

Each of the following is a common side effect of prolonged tetracycline therapy EXCEPT one.
Which one is this EXCEPTION?
F. Diarrhea
G. Superinfection
H. Photosensitivity
I. Visual disturbance
J. Discoloration of newly forming teeth
How do the surface characteristics of a restoration affect its perceived form?
E. A surface smoother than normal will give the impression of a larger size.
F. Increasing the value of the restoration makes the tooth appear smaller.
G. Horizontal highlights give an illusion of increased length.
H. Vertical highlights give an illusion of increased width.
Bilateral enlargement of the parotid glands characterizes each of the following conditions
EXCEPT one. Which one is the EXCEPTION?
F. Malnutrition
G. Sialolithiasis
H. Sjogren's syndrome
I. Benign lymphoepithelial lesion
J. Acute epidemic parotitis
Which of the following best indicates that a removable partial denture needs to be relined?
E. There is a loss of retention.
F. There is soreness on the crest of the ridges.
G. The indirect retainers are not seated as the extension bases are depressed.
H. The acrylic resin teeth are abraded and the occlusal vertical dimension is decreased.

Which of the following is the most common cause of intracapsular restraint of mandibular
movement?
D. Infection
E. Ankylosis
F. Disc interference disorders
A patient will receive a free gingival graft. The graft epithelium will undergo which of the
following alterations?
F. Dysplasia
G. Degeneration
H. Proliferation
I. Orthokeratinization
J. Formation of keratohyalin granules
Which of the following is most likely to cause a yellowish discoloration in a primary maxillary
incisor following trauma?
F. Blood pigments
G. Pulp hyperemia
H. Internal resorption
I. Pulp necrosis
J. Pulp chamber calcification
A patient is allergic to both amide and ester anesthetic derivatives. For this patient, the dentist
should find which of the following infiltrative local anesthetics to be safe and effective?
F. Bupivacaine
G. Phenylephrine
H. Nitrous oxide
I. Diphenhydramine

J. Ethylaminobenzoate
A patient presents with slight chipping of the enamel along the incisal edges of Teeth #8 and #9.
When choosing between restoring the incisal edges or reshaping by selective grinding, which of
the following important factors should the dentist consider?
D. Location of proximal contacts
E. Shape of incisal embrasures
F. Amount of translucent enamel present
Children and adolescents are the LEAST likely to develop which of the following?
E. Ameloblastic fibroma
F. Calcifying epithelial odontogenic tumor
G. Adenomatoid odontogenic tumor
H. Compound odontoma
A dentist is restoring a patient's mandibular arch with a removable partial denture (RPD). The
RPD will replace the second premolars and all molars on both sides. Which of the following is
the best method for recording centric relation?
F. Use a plaster record of the interdigitation of teeth
G. Manually articulate the casts and secure with sticky wax
H. Use the framework that has an occlusal rim attached
I. Use a wax registration that covers premolars of the mandibular arch and anterior teeth
J. Use an occlusal rim made on the master cast to which is added a soft wax for the
registration
The Occupational Safety and Health Administration (OSHA) is concerned with regulated waste
within the office. The Environmental Protection Agency (EPA) regulates the transportation of
waste from the office.
E. Both statements are TRUE.
F. Both statements are FALSE.
G. The first statement is TRUE, the second is FALSE.

H. The first statement is FALSE, the second is TRUE.


A 6-year-old patient has a dark brown discoloration of his maxillary central incisor. The
discoloration started following trauma accompanied with a facial sinus tract. What is the
treatment-of- choice?
E. Extraction
F. Pulpectomy
G. Pulpotomy
H. Direct pulp treatment
Squamous cell carcinoma appears in a variety of locations. In which of the following locations is
its prognosis the LEAST favorable?
E. Lower lip
F. Floor of the mouth
G. Hard palate
H. Buccal mucosa
Although a dentist wishes to use the long-cone paralleling technique (40 cm source-object
distance), the x-ray machine is equipped with only a short cone (20 cm PID). As a result, the
dentist moves the machine away from the patient's face so the source-object distance is 40 cm
and adjusts the exposure time appropriately. Compared to the degree of tissue covered by an
actual long cone, this "extended" short cone exposes what amount of tissue?
D. More
E. The same
F. Less
Fluoride therapy and occlusal sealants modify which of the following four factors the most?
E. Host
F. Time
G. Substrate
H. Microflora

Which of the following drugs produces sufficient central nervous system depression to cause a
state of sleep from which one may be aroused?
E. A sedative
F. A hypnotic
G. An opiate
H. An anesthetic
Which of the following usually causes the slow progress in molar uprighting in an adult patient?
F. Appliance friction
G. Excessive deep bite
H. Occlusal interferences
I. Poor patient cooperation
J. Lack of anchorage control
Squamous cell carcinoma of the tongue MOST commonly metastasizes to which of the
following?
E. Lungs
F. Liver
G. Bones
H. Cervical nodes
To reduce a patient's salivary flow, a dentist has prescribed atropine. As a result of this
medication, the patient might experience which of the following side effects?
F. Sedation
G. Diarrhea
H. Bradycardia
I. Blurred vision
J. Stomach cramping

Which of the following is the OPTIMAL reduction for the lingual cusp on Tooth #3 to receive an
MODL onlay?
E. 1.0 mm
F. 1.5 mm
G. 2.0 mm
H. 2.5 mm
Of the following local anesthetics, which has intrinsic vasoconstrictive actions?
E. Cocaine
F. Procaine
G. Xylocaine
H. Bupivacaine
During an interview, a dentist asks a patient, "You are not afraid of dentistry, are you?" This is an
example of which type of question?
E. Leading
F. Closed
G. Open
H. Funneling
To be effective in translating the roots of teeth, an orthodontic appliance must be
E. very gentle in action.
F. augmented with extraoral force.
G. capable of exerting a torque or moment.
H. capable of exerting positive, intermittent forces.
Which of the following conditions can cause delayed eruption of permanent teeth?
E. Cherubism

F. Hyperthyroidism
G. Hyperparathyroidism
H. Paget's disease of bone
Class I carious lesions are the LEAST likely to occur on which of the following?
E. Lingual surfaces of mandibular incisors
F. Lingual surfaces of maxillary incisors
G. Lingual surfaces of maxillary molars
H. Facial surfaces of mandibular molars
Inhalation of 100% oxygen is contraindicated for a person who has
F. a flail chest.
G. pneumothorax.
H. acute bronchitis.
I. acute viral pneumonia.
J. chronic obstructive lung disease.
Which of the following antibiotics is found at much higher concentrations in crevicular fluid than
in serum?
E. Clindamycin
F. Penicillin
G. Metronidazole
H. Tetracycline
Which of the following exhibits neoplasia of both the epithelial and connective tissue
components?
E. Ameloblastoma
F. Cementoblastoma

G. Ameloblastic fibroma
H. Odontogenic adenomatoid tumor
Non-working interferences usually occur on the inner aspects of the
E. facial cusps of maxillary molars.
F. facial cusps of mandibular molars.
G. lingual cusps of mandibular molars.
H. facial cusps of maxillary premolars.
A vital permanent tooth has an irreversible pulpitis and an open apex. Which of the following
represents the treatment-of-choice for this tooth?
F. Formocresol pulpotomy
G. Indirect pulp therapy
H. Calcium hydroxide pulpotomy
I. Conventional root canal therapy
J. Apexification
Each of the following is associated with Sjogren's syndrome EXCEPT one. Which one is the
EXCEPTION?
F. Arthritis
G. Xerostomia
H. Cervical caries
I. Warthin's tumor
J. Keratoconjunctivitis sicca
A permanent tooth with a mature apex has become intruded deeply into the bone. As a result of
this trauma, which of the following pulpal changes is the most likely to occur?
E. Necrosis
F. Internal resorption

G. Calcific metamorphosis
H. Transient inflammation
Trigeminal neuralgia (tic douloureux) is characterized by
F. paralysis of one side of the face.
G. uncontrollable twitching of one eye.
H. sharp, excruciating pain of short duration.
I. prolonged episodes of pain in one side of the face.
J. dull pain when pressure is applied over the affected area.
An individual has just received the first in a series of three hepatitis B vaccines. For the
seroconversion of this series to be the most effective, the person should receive the second and
third vaccinations how many months later?
E. Second vaccination one month later, third vaccination three months later
F. Second vaccination one month later, third vaccination six months later
G. Second vaccination two months later, third vaccination three months later
H. Second vaccination three months later, third vaccination six months later
Each of the following statements describes intracanal instruments and their action EXCEPT one.
Which one is this EXCEPTION?
E. A K-type file can be used in a filing and reaming action, while a reamer can be used in
only a reaming action.
F. Clockwise-turning of a root canal instrument will force debris in an apical direction,
while counterclockwise-turning will cause debris to move in a coronal direction out of
the canal.
G. A reaming action will enlarge a root canal and produce a cross-sectional preparation that
is approximately round in shape.
H. An intracanal instrument is more susceptible to breakage if locked in dentin and then
rotated.
The usual metabolic pathway of ingested fluoride involves urinary excretion primarily, with the
remaining portion found largely in

E. teeth.
F. muscle tissues.
G. skeletal tissues.
H. epidermal tissues.
Pain that has no organic basis and that is fixed upon some portion of the anatomy is usually
referred to as
E. false.
F. psychogenic.
G. referred.
H. phantom.
In MOST instances, intraoral verruca vulgaris develops as the result of
E. trauma.
F. pipe smoking.
G. autoinoculation.
H. chronic alcoholism.
What happens to the permanent molar occlusion in the presence of a flush (straight) terminal
plane and mandibular primate spaces?
E. Erupts end-to-end; early mesial shift into Class I occlusion
F. Erupts end-to-end; late mesial shift into Class I occlusion
G. Erupts with Class II tendency
H. Erupts with Class III tendency
A patient presents for try-in evaluation of balanced occlusion of complete maxillary and
mandibular dentures. A dentist notes that protrusive excursion results in separation of posterior
teeth. This dentist can best correct this problem by
E. changing the condylar inclination.

F. increasing the incisal guidance.


G. increasing the compensating curve.
H. using a flat plane cusp for the posterior teeth.
Which of the following mucocutaneous disorders is characterized by (1) degeneration of the
basal cell layer, (2) "saw-tooth" rete pegs, and (3) a dense mononuclear inflammatory-cell
infiltrate in the subepithelial connective tissue?
F. Lichen planus
G. Erythema multiforme
H. Keratosis follicularis
I. Dermatitis herpetiformis
J. Benign mucous membrane pemphigoid
An individual's mandible is growing forward less than the maxilla. Each of the following will be
an appropriate treatment strategy EXCEPT one. Which one is this EXCEPTION?
F. Stimulating the mandibular growth
G. Inhibiting the maxillary growth
H. Redirecting the maxillary growth
I. Retracting the mandibular teeth
J. Retracting the maxillary teeth
Nausea and vomiting that are associated with administration of opioid analgesics is the result of
stimulation of the
E. limbic system.
F. emetic center.
G. chemoreceptor trigger zone (CTZ).
H. opioid receptors in the G.I. tract.
Orange stain is used to

E. change the hue of porcelain color.


F. increase the interproximal translucency.
G. increase the value of porcelain color.
H. decrease the chroma of porcelain color.
Vesicular lesions precede the formation of ulcers in each of the following EXCEPT one. Which
one is this EXCEPTION?
F. Herpangina
G. Herpes zoster
H. Herpetic stomatitis
I. Aphthous stomatitis
J. Hand-foot-and-mouth disease
A properly trimmed wooden wedge will do each of the following EXCEPT one. Which one is
this EXCEPTION?
E. Protect the gingival tissue
F. Provide space for the matrix band
G. Prevent overcontouring of the contact area
H. Reduce moisture leakage into the cavity preparation
When is the appropriate time to correct a crossbite of a permanent maxillary central incisor?
E. When the tooth is erupting
F. After the tooth is fully erupted
G. After all anterior teeth are erupted
H. After the opposite central incisor has erupted
Parallelism of abutment preparations is determined by the
F. volume of chamfer.

G. degree of convergence.
H. angulation of finish lines.
I. long axis of the preparations.
J. long axis of natural teeth.
A patient experiences numbness of the left upper lip, cheek, and the left side of the nose
following a fracture of his midface. This symptom follows a fracture through the
E. nasal bone.
F. zygomatic arch.
G. maxillary sinus.
H. infraorbital rim.
When is the appropriate time to take the first bite-wing radiographs of a clinically caries-free
child?
E. When spaces between the posterior teeth have closed
F. When the entire primary dentition is completely erupted
G. When the permanent first molars have erupted
H. At the first dental examination
Which of the following describes primary occlusal trauma?
E. It is the first incidence of trauma that a tooth experiences.
F. It is a trauma that produces irreversible damage to the periodontium.
G. Mobility is caused by excessive forces on a normal periodontium.
H. Mobility is caused by excessive forces on a reduced periodontium.
The maximal or "ceiling" effect of a drug is also correctly referred to as the drug's
E. agonism.
F. potency.

G. efficacy.
H. specificity.
Which condition is characterized by lesions in the central nervous system that manifests itself as
various types of neuromuscular dysfunction?
E. Cerebral palsy
F. Down syndrome
G. A learning disability
H. Hyperkinesis (attention deficit disorder)
In a restorative problem involving all teeth in the mouth, the protrusive condylar path inclination
has a primary influence on the
E. incisal guidance.
F. anterior teeth only.
G. mesial inclines of the mandibular cusps.
H. mesial inclines of the maxillary cusps.
In mg/kg body weight, the lethal dose of fluoride falls in the range of
E. 0.5-1.0.
F. 20-50.
G. 100-200.
H. 300-500.
Which of the following teeth are the least likely to have more than one canal?
F. Maxillary lateral incisors
G. Mandibular lateral incisors
H. Mandibular first premolars
I. Maxillary second premolars

J. Maxillary second molars


Child abuse/neglect most commonly involves children in which age group?
E. Birth to age three years
F. Four to six years
G. Seven to nine years
H. All age groups are equally affected.
A dentist will remove a mandibular lingual torus from a patient's premolar region. This dentist
should use an envelope flap design that
E. has no vertical components.
F. has an anterior vertical component only.
G. has anterior and posterior vertical components.
H. is incised at the junction of attached gingival tissue and free mucosa.
Each of the following will assist a dentist in diagnosing or assessing Sjogren's syndrome
EXCEPT one. Which one is this EXCEPTION?
F. Obtaining sialograms
G. Performing exfoliative cytology
H. Obtaining blood samples for serology
I. Determining salivary flow rate
J. Obtaining labial gland biopsy
The dentist tried-in the metal framework for a porcelain fused-to-metal crown and the margins
were closed. When the completed crown was returned from the lab, the margins are all open. The
most likely reason for this is the
E. die was overtrimmed.
F. lab cut off the margins.
G. casting distorted during the porcelain application.

H. porcelain proximal contact areas are over-contoured


If the line of force were applied through a tooth's center of resistance, the tooth would
F. tip.
G. rotate.
H. intrude.
I. translate.
J. extrude.
Intensifying screens are used with extraoral radiographic films to
E. increase kVp.
F. increase exposure time.
G. improve image quality.
H. decrease radiation to the patient.
An edentulous patient has mobile, hyperplastic tissue in her maxillary anterior region. In making
impressions for this patient, the dentist should
E. use the closed-mouth technique.
F. use a high-fusing impression compound.
G. register the tissue in its passive position.
H. involve maximum pressure.
A patient's permanent tooth crown fractures, creating a small (1 mm) pulp exposure for about 30
minutes. Which of the following pulp therapies is the most appropriate for this patient?
E. Pulpectomy and apexification, if necessary
F. Direct pulp capping with calcium hydroxide
G. Pulpotomy with formocresol
H. Pulpotomy with calcium hydroxide

A 50-year-old man complains of a burning, aching pain on the side of his face. He experiences
paresthesia and itching in the same area. An examination reveals small white scars in the preauricular region. The MOST probable diagnosis is
E. Bell's palsy.
F. psychoneurosis.
G. post-herpetic neuralgia.
H. auriculotemporal syndrome.
Prolonged, unstimulated night pain suggests which of the following conditions of the pulp?
E. Pulp necrosis
F. Mild hyperemia
G. Reversible pulpitis
H. No specific condition
Some teeth appear to be clinically normal, but exhibit (1 ) globular dentin, (2) very early pulpal
obliteration, (3) defective root formation, (4) periapical granulomas and cysts, and (5) premature
exfoliation. The condition is known as which of the following?
F. Shell teeth
G. Dentin dysplasia
H. Regional odontodysplasia
I. Amelogenesis imperfecta
J. Dentinogenesis imperfecta
When a nonrigid connector is used in a fixed partial denture, the path of insertion of the key into
the keyway should be parallel to the path(s) of insertion of
D. the retainer carrying the keyway.
E. the retainer not involved with the keyway.
F. both retainers.
Green and orange stains on maxillary incisors can usually be attributed to

E. drugs.
F. diet.
G. poor oral hygiene.
H. fluoride consumption.
The best measure of the potential clinical performance of a casting alloy is its
F. castability.
G. burnishability.
H. ADA certification.
I. tarnish susceptibility.
J. mechanical properties.
Which of the following cysts is the LEAST likely to be visible radiographically?
E. Nasolabial
F. Nasopalatine
G. Lateral periodontal
H. Traumatic bone
Which drug group is the least likely to cause xerostomia?
F. Opioids
G. Antidepressants
H. Antihistaminics
I. Benzodiazepines
J. Anticholinergics
Which of the following is measured by the modulus of elasticity?
E. Stiffness or rigidity

F. Ultimate strength
G. Yield strength
H. Ductility or malleability
Which of the following represent(s) the serious effects of a repeated exposure to low doses of X
radiation?
E. Purpura
F. Carcinogenesis
G. Diarrhea and dehydration
H. Alteration of the oral microflora
Currently, the best oral sedative drugs for dentistry fall into the class of
E. narcotics.
F. barbiturates.
G. phenothiazines.
H. benzodiazepines.
Which of the following best describes the drug-receptor activity of naloxone?
E. High affinity, high intrinsic activity
F. Low affinity, high intrinsic activity
G. High affinity, no intrinsic activity
H. No affinity, low intrinsic activity
The amalgam preparation outline for the primary mandibular second molar closely resembles
that for which permanent tooth?
E. Maxillary second premolar
F. Mandibular first premolar
G. Mandibular second premolar

H. Mandibular first molar


The rate at which new disease occurs is classified as which of the following?
E. Incidence
F. Prevalence
G. Extent
H. Attributable risk
Which of the following conditions indicates that a periodontal, rather than an endodontic
problem, exists?
E. Acute pain to percussion with no swelling
F. Pain to lateral percussion with a wide sulcular pocket
G. A deep narrow sulcular pocket to the apex with exudate
H. Pain to palpation of the buccal mucosa near the tooth ape
An 82-year-old woman presents with a large four-surface pin-retained amalgam restoration on
Tooth #3. The dentist notes minor recurrent caries along the faciocervical amalgam margin.
Which of the following is the treatment-of-choice?
E. Replace the restoration
F. Repair the defect
G. Prepare the tooth for a crown
H. Observe at recall
The proximal portion of a Class II cavity preparation in a primary molar extends rather deep
gingivally. A satisfactory gingival seat may NOT be obtained because of which of the following?
E. The primary teeth have a marked cervical constriction.
F. The proximal contact of primary molars is broad and flat.
G. The facial and lingual surfaces of primary molars converge occlusally.
H. The enamel rods in the gingival third of the primary first molars extend occlusally.

During which stage of tooth development is the cariostatic effect of fluoride manifested?
E. Apposition
F. Calcification
G. Proliferation
H. Histodifferentiation
The light source affects the perception of color, BECAUSE the light source must contain the
wavelength of the color to be matched in order to see that color.
F. Both the statement and the reason are correct and related.
G. Both the statement and the reason are correct but NOT related.
H. The statement is correct, but the reason is NOT.
I. The statement is NOT correct, but the reason is accurate.
J. NEITHER the statement NOR the reason is correct.
Orthodontic correction of which of the following is the most easily retained?
F. Diastema
G. Rotation
H. Expansion
I. Anterior crossbite
J. Generalized spacing
Which of the following types of chemical bonding is the least likely to be involved in a drugreceptor interaction?
F. Covalent bonding
G. Hydrogen bonding
H. Dipole-dipole bonding
I. Electrostatic bonding

J. van der Waal's forces


Which of the following best explains why the renal excretion of salicylic acid is increased in an
alkaline urine?
E. The urine volume is decreased.
F. The percentage of salicylic acid that is ionized in urine is increased.
G. The percentage of salicylic acid that is ionized in urine is decreased.
H. Decreasing urine pH usually increases the renal excretion of weak acids.
X-rays are produced when
E. protons strike the anode.
F. electrons strike the anode.
G. the anode is heated above 3,000 degrees Centigrade.
H. the filament becomes positively charged.
A member of the dental staff routinely fails to clean up his work area. Which of the following
best exemplifies an assertive message that could be made by a co-worker?
F. If you refuse to do your part, I'm just going to have to report the situation to the boss.
G. When you don't help with the clean-up, it makes me angry because I have to do both your
work and my own.
H. I'm tired of cleaning up your mess!
I. I hope you'll remember to help me clean up today.
J. Is something bothering you? You sometimes forget to clean up.
The crown of an endodontically-treated maxillary lateral incisor is fractured near the gingival
margin. The coronal end of the silver cone used in filling that canal is visible at that level. The
findings reveal that the existing root canal filling meets all criteria to be judged successful.
Which of the following is the best way to obtain the needed post space?
E. To prepare it alongside the silver cone using burs and Peeso reamers.
F. To carefully grind away the coronal part of the silver cone using round burs or endcutting burs.

G. To remove the silver cone and re-treat the canal using a gutta-percha technique, and then
to create the needed post space.
H. To remove the silver cone, notch it 4 mm from the apical end, coat it with freshly mixed
sealer, replace it in the canal, and "twist-off" the coronal segment.
After receiving one cartridge of a local anesthetic, a healthy adult patient became unconscious in
the dental chair. The occurrence of a brief convulsion is
E. pathognomonic of grand mal epilepsy.
F. consistent with a diagnosis of syncope.
G. usually caused by the epinephrine in the local anesthetic.
H. pathognomonic of intravascular injection of a local anesthetic
The axial wall of an occlusolingual amalgam preparation on a maxillary molar should be in
dentin and
D. parallel to the dentinoenamel junction.
E. parallel to the long axis of the tooth.
F. at an acute angle with the pulpal floor.
During the mixed dentition stage, which of the following appliances should be used as a space
maintainer for missing primary molars in the mandibular arch?
E. Distal shoe
F. Nance holding arch
G. Passive lingual arch
H. Removable acrylic functional
Accepted methods for closing a diastema between maxillary central incisors include using either
a removable appliance with finger springs or
D. a rubber elastic around the two teeth.
E. a steel ligature around the two teeth.
F. bonded brackets with intertooth traction.

Each of the following developmental cysts appears on radiographs EXCEPT one. Which one is
this EXCEPTION?
E. Nasolabial
F. Median palatal
G. Globulomaxillary
H. Nasopalatine
When trying-in a porcelain fused-to-metal crown, the dentist observes that the gingival-margin
finish-line integrity is excellent, but that the occlusal surface is 1 mm too high. Which of the
following is the most probable cause?
E. Incorrectly related casts
F. Proximal contacts being too tight
G. A distortion of the metal during firing
H. An expansion of the porcelain during firing
The Health Belief Model attempts to explain patient recommendations by
E. examining patients' perceptions of disease-seriousness and of the treatment effectiveness.
F. focusing on the doctor-patient relationship.
G. emphasizing the barriers that are external to behavior change, for example, payment
mechanisms.
H. focusing on patients' skills at carrying out recommendations.
Which of the following represents the MOST effective way of encouraging a child to floss
regularly?
E. Presenting a message that concerns the potential loss of teeth, should the patient not floss
F. Arranging for the child to earn privileges for flossing
G. Repeating periodically the message on the value of flossing
H. Providing written material that explains the mechanism of flossing

In an ideal Class V cavity preparation for amalgam in a mandibular premolar, retention form is
gained
E. into the mesial and distal walls.
F. in the mesial and occlusal line angles.
G. in the axio-occlusal and axiogingival line angles at the expense of the axial wall.
H. in the axio-occlusal and axiogingival line angles at the expense of the occlusal and
gingival walls.
Which of the following systems is thought to malfunction in the hereditary form of angioneurotic
edema?
F. C-1 esterase
G. C-1q inhibitor
H. CH50 consumption
I. Serine phosphatase
J. Complement synthetase
Following a bilateral mandibular fracture in the canine region, the anterior fragment of the
mandible is displaced posteriorly by the action of the geniohyoid and by which other muscles?
E. Thyrohyoid and mylohyoid only
F. Genioglossus and mylohyoid
G. Thyrohyoid, mylohyoid, and anterior bellies of the digastric
H. Genioglossus and anterior bellies of the digastric
The number of persons who die each year from oral cancer in the United States approximates
E. 2000
F. 4000
G. 8000
H. 16000

Each of the following parameters has an effect on the distribution of a drug EXCEPT one. Which
one is this EXCEPTION?
E. The intrinsic activity of the drug
F. The binding of a drug to plasma proteins
G. The blood flow to various organs and tissues
H. The relative solubility of the drug in the tissues
Which of the following is MOST appropriate for testing differences between the means of two
groups?
E. Chi-square test
F. Multiple regression analysis
G. Correlation coefficient analysis
H. Student's t-test
When a removable partial denture is terminally seated, the retentive clasp tips should
E. be invisible.
F. exert no force.
G. apply retentive force into the body of the teeth.
H. resist torque through the long axis of the teeth.
The decrease in ventilation caused by morphine and by some of the related opioids depends
chiefly upon a decrease in
E. blood PC02 concentration.
F. medullary endorphin activity.
G. sensitivity of the medullary respiratory center to carbon dioxide.
H. sensitivity of aortic and carotid chemoreceptors to blood oxygen concentration.
The depth of the clinical gingival sulcus is the distance from the gingival margin to the
E. cementoenamel junction.

F. most apical penetration of the periodontal probe.


G. most apical extension of the junctional epithelium.
H. bottom of the sulcus in well-preserved histologic block sections.
Additional space for successive eruption of permanent maxillary molars is provided by
E. interstitial bone growth.
F. appositional growth at the maxillary tuberosity.
G. continuous expansion of the dental arch due to sutural growth.
H. an increase in palatal vault height due to alveolar growth.
Which of the following responses made by the dentist is the MOST likely to increase a patient's
adherence to oral hygiene prescriptions?
F. You don't want lots of cavities, do you?
G. If you take my advice, you can save yourself pain and money.
H. You really should spend the time it takes to brush and floss.
I. What do you see as some of the problems with this approach?
J. I can assure you that this approach is quick and easy.
Amalgam scrap should be stored in a tightly-sealed container and covered with which of the
following?
E. Water
F. Glycerin
G. Sulfide solution
H. Sodium hypochlorite solution
The soft tissue-tooth interface that forms most frequently after flap surgery in an area previously
denuded by inflammatory disease is a
E. collagen adhesion.
F. reattachment by scar.

G. long junctional epithelium.


H. connective tissue attachment
Which of the following is inherited as an autosomal dominant trait?
F. Lichen planus
G. Bullous pemphigoid
H. Pemphigus vulgaris
I. White sponge nevus
J. Epidermolysis bullosa acquisita
Which of the following is the primary substance or material removed during root planing?
F. Stain
G. Dentin
H. Plaque
I. Calculus
J. Cementum
Three days after full mouth extraction, an elderly patient complains of black-and-blue marks on
her neck. The most probable diagnosis is
E. thrombocytopenia.
F. capillary fragility.
G. hematoma formation.
H. postoperative ecchymosis.
Most epidemiologic studies indicate that gingivitis in children is relatively common. A strong
positive association between specific nutritional deficiencies and the presence of periodontal
disease in children and adults has been demonstrated.
E. Both statements are TRUE.
F. Both statements are FALSE.

G. The first statement is TRUE, the second is FALSE.


H. The first statement is FALSE, the second is TRUE.
Chroma is that aspect of color that signifies the
E. degree of grayness.
F. degree of translucency.
G. degree of saturation of the hue.
H. combined effect of the hue and value.
Which of the following characterize(s) the histopathologic changes in chronic gingivitis?
F. The loss of rete pegs and the dissolution of the basement membrane
G. An inflammatory infiltrate in which macrophages predominate
H. An inflammatory infiltrate in which neutrophilic leukocytes and mast cells predominate
I. The destruction of the principal fibers of the periodontal ligament
J. The disruption of the gingival fibers and an inflammatory infiltrate of plasma cells,
lymphocytes, and neutrophilic leukocytes
Which of the following most strongly suggest cause-and-effect relationships?
E. Correlational studies
F. Controlled clinical trials
G. Case reports
H. Epidemiologic surveys
Spontaneous osteogenic sarcoma is associated with which of the following disease processes of
bone?
F. Paget's disease
G. Histiocytosis-X
H. Hyperparathyroidism

I. Letterer-Siwe disease
J. Osteogenesis imperfecta
The drug-of-choice for the treatment of adrenergically-induced arrhythmias is
E. quinidine.
F. lidocaine.
G. phenytoin.
H. propranolol.
Which of the following drugs, when administered intravenously, is LEAST likely to produce
respiratory depression?
F. Fentanyl
G. Diazepam
H. Thiopental
I. Meperidine
J. Pentobarbital
Which of the following safety techniques provides the GREATEST DECREASE in overall
radiation-risk to patients?
E. Changing from Group D to Group E film
F. Switching from round to rectangular collimation
G. Using an automatic rather than manual processing switch
H. Adding a cervical collar to a leaded apron
Premature exfoliation of a primary mandibular canine is most often the sequela of which of the
following?
E. Caries
F. Trauma
G. Serial tooth extraction

H. Arch length inadequacy


In lateral movements, the non-working condyle moves in what direction?
E. Laterally, posteriorly, and superiorly
F. Superiorly, posteriorly, and inferiorly
G. Inferiorly, anteriorly, and medially
H. Inferiorly, anteriorly, and laterally
A dentist who is planning to restore a tooth with gold alloy desires a certain amount of expansion
in the casting investment mold. Which of the following best explains why this expansion is
desirable?
D. It allows for the shrinkage of the molten metal and the wax pattern.
E. It compensates for the shrinkage of the molten metal as it cools in the mold.
F. It makes the metal casting larger than the wax pattern so that the casting will fit the tooth
tightly.
The dentist is using polyether material to make impressions. The physical property of this
material that should concern this dentist is the
E. coefficient of thermal conductivity.
F. hydrophilic nature.
G. extended setting time.
H. high modulus of elasticity.
Streptococcus mutans is considered to be a principal etiologic agent of caries because it produces
organic acids and it
E. forms a gelatinous matrix.
F. metabolizes substrate from saliva.
G. derives energy from enamel constituents.
H. lives symbiotically with Lactobacillus acidophilus.

Which of the following best explains how high-copper amalgam restorations differ from
conventional amalgam restorations? The high-copper restorations
E. have little or no tin-mercury phase.
F. require less mercury, so there is more matrix formed.
G. corrode at an accelerated rate due to increased copper content.
H. are unaffected by moisture contamination in the presence of zinc
What do Gardner syndrome and cleidocranial dysplasia have in common?
E. Intestinal polyps
F. Intraoral pigmentation
G. Impacted supernumerary teeth
H. Osteomas of the skull and jaws
A practitioner who is restoring a tooth with composites wishes to be sure that the matrix has
adhered to the filler. Which of the following agents adheres the resin matrix to the filler?
E. Wetting
F. Coupling
G. Catalyzing
H. Activating
An operator has chosen to use a shielded open-ended cone. Which of the following will
contribute the most to patient gonadal dose?
E. Leakage from the x-ray machine head
F. Scatter from the operatory walls
G. Scatter from the cone
H. Scatter from the patient's face
Which of the following provides the best guarantee for sterilization in a heat sterilizer?
F. Using a chemical indicator strip or pouch

G. Recording the temperature and/or pressure readings from the sterilizer gauges
H. Using bacterial spore tests
I. Determining the ability of the sterilizer to kill the hepatitis B virus
J. Demonstrating inactivation of the tuberculosis bacterium
The integrity of the floor of the antrum is at greatest risk with surgery involving the removal of
(a)
F. torus palatinus.
G. nasopalatine cyst.
H. maxillary third molar.
I. single remaining maxillary molar.
J. supernumerary teeth from the maxillary canine region.
Which of the following is the principal nonverbal cue that two or more people can use to regulate
verbal communication?
E. Posture
F. Movement
G. Eye contact
H. Body position
When the mandible is in its physiologic rest position, the distance between the occluding
surfaces of the maxillary and mandibular teeth or occlusion rims is
E. vertical dimension.
F. interocclusal distance.
G. vertical dimension of rest.
H. centric occlusion (maximum intercuspation).
The practitioner would like to learn whether or not a patient understands the treatment plan.
Which of the following questions is the MOST likely to elicit this information?

E. Do you understand what I mean?


F. Do you have any questions about this approach?
G. What benefits or drawbacks do you see in this approach?
H. Is this approach OK with you?
The distolingual extension of a mandibular impression for a complete denture is limited by the
action of which of the following muscles?
E. Stylohyoid
F. Lateral pterygoid
G. Medial pterygoid
H. Superior constrictor
A defect in neuromuscular transmission causes which of the following?
F. Bell's palsy
G. Myasthenia gravis
H. Muscular dystrophy
I. Multiple sclerosis
J. Trigeminal neuralgia
The day after receiving an inferior alveolar nerve block, a patient experiences limited ability to
open his mouth. Which of the following structures was most probably injured?
F. The medial pterygoid muscle
G. The stylomandibular ligament
H. The deep fibers of the masseter muscle
I. The posterior belly of the digastric muscle
J. The inferior head of the lateral pterygoid muscle
Lesions of recurrent herpetic stomatitis are usually found mainly on

F. loosely attached areas of the oral mucosa, such as the buccal mucosa and the floor of the
mouth.
G. tightly attached areas of the oral mucosa, such as the hard palate and the attached gingiva.
H. loosely attached areas of the oral mucosa, such as the soft palate and
I. the alveolar mucosa.
J. tissue anywhere in the oral mucosa, including both loosely- and tightly-attached areas.
Which of the following instruments should be used to plane the facio-proximal cavosurface
margin of a standard Class II preparation on a mandibular molar?
E. Straight chisel
F. Binangle chisel
G. Enamel hatchet
H. Bibeveled hatchet
Which of the following most accurately describes the effect of individual plaque control
instructions that are conducted in the classroom?
E. A short-term decrease in gingivitis
F. A long-term decrease in gingivitis
G. A short-term decrease in caries
H. A long-term decrease in caries
An endodontic instrument separated in the apical third of a root canal. The fragment is 3 mm
long and is tightly lodged. No radiographic changes at the apex are evident. The practitioner
should
E. extract the tooth.
F. resect the apical section of the root containing the broken instrument.
G. perform an apicoectomy and place a reverse filling.
H. complete the root canal filling to the level of the instrument and observe.

A new patient had root canal therapy performed seven months ago in another country. No
historical radiographs are available. The root canal filling appears to be satisfactory, the tooth is
asymptomatic, and there is no associated sinus tract. However, a small periapical radiolucency is
evident. Which of the following is indicated?
F. Incision and drainage
G. Nonsurgical retreatment
H. Re-evaluation in six months
I. Apicoectomy and apical amalgam
J. Prescription of an appropriate antibiotic
Which of the following do polycarboxylate and glass ionomer have in common?
F. Zinc oxide
G. Polysiloxane
H. Phosphoric acid
I. Polyacrylic acid
J. Ion-leachable glass
A practitioner has burnished a margin. As a result of this procedure, the dental casting gold has
become stronger, but less ductile. Which of the following explains why?
E. The gold has work-hardened.
F. The grains have elongated.
G. The stiffness has increased.
H. The surface flaws have been removed.
The light from visible-fight polymerization units can cause which of the following?
E. Iritis
F. Cataracts
G. Retinal damage

H. Corneal ulcerations
Which of the following responses made to a child is MOST likely to reinforce positively the
child's appropriate behavior in the dental setting?
E. You're doing a great job for me today.
F. I like it when you sit quietly in the chair.
G. This visit is going really well, don't you think?
H. Coming to the dentist isn't so bad after all, is it?
An endodontically-treated permanent mandibular first molar has incipient lesions on its mesial
and distal surfaces. During previous treatment, a minimal amount of tooth structure was
removed. The appropriate treatment for this tooth is a (an)
F. MOD amalgam.
G. MOD cast gold inlay.
H. MOD cast gold onlay.
I. 3/4 crown.
J. full crown.
A patient has a skeletal deformity with a Class III malocclusion. This deformity is the result of a
maxillary deficiency. The treatment-of -choice is
F. orthodontics.
G. surgical repositioning of the maxilla.
H. anterior maxillary osteotomy.
I. posterior maxillary osteotomy.
J. surgical repositioning of the mandible.

Das könnte Ihnen auch gefallen